You are on page 1of 43

‫اسئلة وزارة الصحة لاللتحاق‬

‫ببرنامج االقامة واالمتياز‬


‫( ‪) 2018 -2013‬‬

‫بفضل هللا قمت باعادة جمع وتنسيق اسئلة‬


‫الدورات السابقة‪ ,‬الصادرة من وزارة الصحة‬

‫*****‬
‫المتحانـــات االقامـــة واالمتيــاز لالعـــوام‬
‫( ‪ , ) 2018 -2013‬بعد جهـــد وعنــاء ‪,‬‬
‫وحلها باالستعانة بالمراجع والكتب المختلفة‬

‫‪PEDIATRIC‬‬
‫لتوخي الدقة في نشر المعلومة الصحيحــة‬
‫للزمالء والزميالت ‪ ,‬فإن ( احسنت فمن هللا ‪,‬‬
‫وان اسأت فمن نفسي ) ‪ ,‬واني اقـــدم هذا‬
‫العمل خالصا لوجه هللا تعالى ‪ ,‬راجيا المولى‬

‫*****‬
‫القبول ‪ ,‬وان ينال اعجابكم ‪.‬‬

‫اللهم استودعك ما قرأت وما حفظت‬


‫وتعلمـت ‪ ،‬فـرده لـي عنـد حاجتـي اليه ‪,‬‬
‫إنـــك على ما تشاء قدير‬

‫نسخة مدققة‬
‫اخوكم الدكتور ‪ :‬اسامـه الخزاعلـه‬
‫‪0799430239‬‬
‫طـــب عـــام وجراحــه‬
* The arterial blood gas of a B . Respiratory acidosis
comatose infant is as follows : PH = C . Metabolic acidosis
7.45 , PaCO2 = 25 , PaO2 = 90 , HCO3 D . Metabolic alkalosis
= 12 . Which of the following best E . Mixed alkalosis
describes these numbers ?
A . Metabolic acidosis
B . Metabolic alkalosis
C . Respiratory acidosis
D . Respiratory alkalosis * Which of the following is not
E . Combined respiratory and associated with prolonged bleeding
metabolic acidosis time ?
A . Uremia
* A normal infant sits without B . Thrombocytopenia
support easily and can walk while C . Hemophilia
holding on to furniture , her age is D . VWD
A . 4 months E . Glanzmann disease
B . 9 months
C . 14 months * What is the most common cause of
D . 18 months IDA in Jordan ?
A . Gastrointestinal parasitic
* In regards to trisomy 21 ( Down infection .
syndrome ) which of the following is B . Drinking tea after each meal .
true ? C . Inadequate exposure to the sun .
A . Only affects children born to D . Family eats too much fast food .
very old mother . E . Consumption of large amount of
B . Most common feature is cow's milk
hypertonia .
C . There is a 50 % risk the child * A 6 year old child presents with
will have congenital heart disease . fever and sore throat . Exam shows
D . The older the father the more red throat without enlarged neck
likely he is Down syndrome . lymph nodes . The next step in
E . Social behavior is delayed for management is ?
age . A . Start amoxicillin for 7 days .
B . Start Azithromycin for 7 days .
* Most common solid tumor for C . Paracetamol for pain and follow
children is ? up in second
A . CNS tumor D . Refer to ENT specialist .
B . Neuroblastoma E . Obtain WBC and blood culture and
C . Willms tumor make decision after result .
D . Bone tumor
E . Retinoblastoma * The most common cause of
convulsion in a 2 years old infant
* In DKA all are true except ? is ?
A . Insulin should not be given A . Grand mal seizure
until check the serum potassium . B . Petit mal seizure
B . KCL usually given when serum C . Febrile convulsion
potassium is normal or low . D . Brain tumor
C . Mannitol is given when E . Ruptured aneurysm
bicarbonate HCO3 is very low .
D . NaCl solution is the best choice * One of the following is included
. in Jordan nation immunization
E . Sodium bicarbonate should be program ?
given if PH less than 7 . A . Pneumococcus
B . Rota virus
* The ABG of a child on ventilator C . Meningococcus
shows : PH = 7.5 , PaO2 = 90 , PaCO2 D . Varicella
= 20 , HCO3 = 22 . Which of the E . Hepatitis A
following best describes these
numbers ?
A . Respiratory alkalosis

‫اسئلــة االطفـــال‬ ‫ اسامه الخزاعله‬: ‫الدكتور‬


Page 2
* Regarding the treatment of A . Dog bites generally get infected
jaundice , which of the following is faster than cat bites .
correct ? B . Amoxicillin clavulanic acid is
A . The use of phototherapy has no an excellent option for bites .
complications . C . Dog and cat bites should be
B . Premature babies have a higher treated with ATS while human bites
risk for jaundice . can be observed .
C . Physiologic jaundice results D . All children who have bitten
from the elevation of conjugated should received antibiotics
bilirubin . intravenously .
D . Jaundice in breastfeed babies E . The treatment of bites from
occurring at day 6_9 of life should pet's is observed , while from stray
managed by increased breastfeeding . animals should be admitted to the
E . Jaundice in breastfeed babies hospital .
occurring at day 1_3 of life should
managed by discontinuing * Charcoal is used in ingestion of
breastfeeding for a few days one of the following substances ?
A . Iron
* A child is able to walk down B . Hydrocarbon
stairs holding the side rail , throw C . Corrosive materials
a ball over hand , and initiate D . Lead
drawing a horizontal line , he could E . Carbamazepine
say 2 words
sentence , he is most likely ?
A . 24 months
B . 30 months
C . 36 months
D . 48 months * All are contraindications to
E . 60 months breastfeeding except ?
A . Maternal drug abuse .
* 10 day old boy presented with B . Mother receiving chemotherapy .
vomiting , dehydration , decreased C . Maternal hepatitis .
feeding and hypotension . He was D . Maternal active tuberculosis .
found to have serum sodium = 118 , E . Infant diagnosed with
potassium = 7 , glucose = 65 . The Galactosemia .
best management for this boy is to
give ? * vitamin D supplementation is
A . Antibiotics and IV fluid recommended in infant from the age
B . Hydrocortisone and normal saline of ?
C . Adrenaline sc A . After birth
D . Normal saline B . One month
E . Sodium bicarbonate and calcium C . Three months
gluconate D . Six months
E . One year
* A 9 year old Jordanian female
presented with recurrent episodes * The following is an acceptable
and abdominal pain for few years . alternative for an infant with cow
She has history of multiple milk protein intolerance except ?
hospitalization . Her father has A . Hydrolyzed formula
also same scenario . The most likely B . Amino acid formula
diagnosis is ? C . Lactose free formula
A . FMF D . Soy formula
B . Appendicitis E . Breast milk
C . Cholycystitis
D . UTI * All of the following are
E . Diverticulitis characteristic of a PDA except ?
A . Continuous machinery murmur .
* In regards to bites , which of the B . Narrow pulse pressure .
following is correct ? C . Prominent pulmonary artery on
CXR .

‫اسئلــة االطفـــال‬ ‫ اسامه الخزاعله‬: ‫الدكتور‬


Page 3
D . Risk of infective endocarditis . B . Child is consistently below the
E . Can cause pulmonary hypertension 5th percentile for age on height ,
. weight charts .
C . Child was exclusively breastfeed
* Regarding juvenile rheumatoid until age 6 months .
arthritis , one is false ? D . Child drinks a lot of fruit
A . The most common type is the juice .
pauciarticular . E . Child is playful and active .
B . More common in females .
C . Rheumatoid factor is positive in * All are causes of immunodeficiency
most of the case's . except ?
D . NSAIDs and methotrexate are used A . Di_George syndrome
in treatment . B . Wiskott Aldrich syndrome
E . May be complicated by C . Ataxia telangiectasia
iridocyclitis or uveitis . D . Phenylketonuria
E . Hyper IgE syndrome
* A 12 year old boy brought to the
E.R with wheezing , difficulty of * Although all of the following
breathing and hives . The BP 90/60 , medications are used in treatment of
HR 140 . His pulse ox was 88% . The Hyperkalemia in children . The only
father stated he is ingested Tuna 15 one that can remove the potassium
minutes before his symptoms from the body is ?
started . The patient is known to A . 10% calcium gluconate
have multiple food allergies . Of B . Bicarbonate
the following the most appropriate C . Nebulized albuterol
immediate action is ? D . Insulin
A . Administration of oxygen . E . Kayexalate
B . Administration of NaCl saline
bolus . * A 3 weeks old infant is admitted
C . Administration of epinephrine . with vomiting of 5 days duration ,
D . Administration of beta 2 agonist physical exam reveals a rapid heart
Nebulizer . rate , evidence of dehydration ,
E . Administration of steroid and ambiguous
antihistamine . genitalia . Na= 120 , k =7 ,HCO3 =
12 . The best next step is ?
A . Diuretics
B . Potassium exchange resin
* Which of the following would you C . Glucose and insulin
expect in a child with carbon D . Antibiotics
monoxide poisoning ? E . Hydrocortisone and normal saline
A . Bradycardia
B . Metabolic alkalosis
C . Headache , nausea and vomiting
D . HTN
E . Bradypnea
* An 18 year old boy presents with
* All of the following are causes of cough , chest pain and low grade
Corrigan pulse except ? nightly fever for several weeks .
A . Fever CXR reveals a large mass in the
B . Aortic stenosis mediastinum . Most likely diagnosis
C . Beriberi is ?
D . Paget disease A. Adenocarcinoma
E . Cor pulmonale B . SCC
C . Small cell carcinoma
* Which of the following is most D . Lymphoma
concerning for failure to thrive in E . Metastatic Willms tumor
a child ?
A . Child does not finish his plate * A 3 day old term , healthy infant
during meal . is noted to be jaundiced . Physical
exam is otherwise normal ,

‫اسئلــة االطفـــال‬ ‫ اسامه الخزاعله‬: ‫الدكتور‬


Page 4
laboratory HB 16.8 g/dL ,
Reticulocytes 1% , unconjugated * Acute bronchiolitis characterized
bilirubin 8.5 mg /dL , conjugated by one of the following ?
0.8 mg dL . The most likely A . Usually associated with high
diagnosis is ? fever .
A . Breastfeeding jaundice B . Usually associated with lobar
B . Breast milk jaundice infiltrates on CXR .
C . Physiological jaundice C . Commonly associated with
D . Crigler najjar syndrome retractions , tachypnea and wheezing
E . Erythroblastosis .
D . Absent of cough .
* Over the first week of life , a E . Most common between 2 _ 5 years
typical term newborn will ? age
A . Gain approximately 30 g per
day . * An infant with diarrhea is 10%
B . Gain approximately 60 g per dehydrated . The weight was 5 kg .
day . What is the total amount of fluid
C . Neither gain nor lose weight . you should deliver to meet
D . Lose 10% of it's birth weight . maintenance needs and restore to
E . Lose 15% of it's birth weight . normal hydration state in the first
24 hours ?
* Overdose of which of the following A . 750 ml
is most likely to be complicated by B . 1000 ml
hypoglycemia ? C . 1250 ml
A . Salicylates D . 1500 ml
B . Lead E . 1750 ml
C . Tricyclic antidepressants
D . Opiods * Pyruvate kinase deficiency is the
E . Organophosphates most common glycolytic enzyme
deficiency . Which of the following
* The recommended daily intake of is a common clinical sign in
protein for optimal growth during neonatal
the first 6 months of life is presentation of this disorder ?
about ? A . Sepsis and infections
A . 0.2 g/kg B . Jaundice
B . 1 g/kg C . Hepatomegaly
C . 2 g/kg D . Hypertonia
D . 5 g/kg E . Seizures
E . 10 g/kg
Which of the following is the most
* Delayed dental eruption is common clinical manifestations that
considered when the child has no follows infection with Campylobacter
teeth by age ? ?
A . 6 months A . Polyarticular arthritis
B . 8 months B . Encephalitis
C . 11 months C . Guillain barre syndrome
D . 13 months D . Inguinal lymphadenitis
E . 18 months E . Anterior uveitis

* Which of the following is


characteristic of the CSF in
tuberculous meningitis ?
A . The color is blood tinged .
* The tonic neck reflex should B . Protein is normal .
disappear by the age ? C . Culture TB organisms within one
A . 3 months week .
B . 4 months D . Glucose is low .
C . 6 _ 8 months E . Leucocytes predominant .
D . 12 _ 16 months
E . 17 _ 20 months

‫اسئلــة االطفـــال‬ ‫ اسامه الخزاعله‬: ‫الدكتور‬


Page 5
* The diagnosis of Werding Hoffman
disease is most likely in an infant
with severe hypotonia and
A . Normal deep tendon reflexes .
B . Seizures . * After your intervention whole
C . Fasciculations of the tongue . blood glucose is 500 mg /dL .
D . Recurrent fever . Intravenous fluid and insulin are
E . Atrophy of the optic nerve . given . Which of the following
metabolic abnormalities is most
* An 8 months old child has vomiting likely to occur during insulin
and scream episodes for 12 hours . therapy ?
Physical exam reveal a sausage A . Hyperkalemia
shaped mass in the RUQ . Which of B . Hypokalemia
the following would be most useful ? C . Hyperphosphatemia
A . Passage of NG tube . D . Hypercalcemia
B . Exam of stool specimen for ova E . Hypermagnesemia
and parasites
C . Blood culture . * Patient with Down syndrome are at
D . Abdominal ultrasound . increased risk of ?
E . Barium enema study . A . Hyperthyroidism
B . Arthritis of cervical spine
* Complex partial ( psychomotor ) C . Streak gonads
seizures are characterized by ? D . Cardiac malformation
A .Lack of alternation in mental E . Rhabdomyosarcoma
state , consciousness,
responsiveness . * In patient with cystic fibrosis ,
B . Brief tonic clonic phase . all are expected except ?
C . Automatisms . A . Rectal prolapse .
D . Three per second spike on EEG . B . Nasal polyp .
E . Lack of postictal phenomenon . C . Cor pulmonale .
D . Coronary artery aneurysm .
* Rectal prolapse in an infant with E . Azoospermia .
failure to thrive is most suggestive
of ? * One is true about caustic
A . Crohn disease ingestion ?
B . Functional constipation A . Early endoscopy is indication .
C . Ehlers Danlos syndrome B . Induce vomiting reduce the
D . Cystic fibrosis complications .
E . Intussusception C . Absence of the mouth lesions
exclude the esophageal injury .
* Which of the following is most D . Gastric lavage is
appropriate for the treatment of contraindicated .
Mycoplasma pneumonia infection in a E . Acidotic agent's is more harmful
4 year old child ? than the alkaline agent's .
A . Cefuroxime
B . Chloramphenicol * All of the following could be
C . Erythromycin options for treating brucella except
D . Penicillin ?
E . Tetracycline A . Doxycycline
B . Rifampin
* Neurotoxicity is a common adverse C . Gentamicin
effect of which chemotherapy ? D . Trimethoprim / Sulfamethoxazole
A . Methotrexate E . Cefazolin
B . Cyclophosphamide
C . Chlorambucil * All of the following are
D . Bleomycin recognized causes of hemolytic
E . Vincristine disease in the newborn except ?
A . Hereditary spherocytosis .
B . G6PD deficiency .

‫اسئلــة االطفـــال‬ ‫ اسامه الخزاعله‬: ‫الدكتور‬


Page 6
C . Sickle cell disease . A . It is included in the Jordanian
D . ABO incompatibility . national vaccination program .
E . RH incompatibility . B . It has little effect on the
reactivation disease .
* The daily IV fluid maintenance for C . May affect tuberculosis skin
a 15 kg child is ? test ( PPD ) result .
A . 750 cc . D . Can be given from birth .
B . 1250 cc . E . It is a killed bacterial vaccine
C . 1900 cc . .
D . 2500 cc .
E . 3000 cc . * Acrodermatitis enteropathica is
* Which of the following set's of treated with ?
signs and symptoms are most A . Vitamin A .
consistent with mild dehydration 5% B . Vitamin B2 .
in infancy ? C . Phosphate .
A . Oliguria , tears with crying , D . Magnesium .
less active than usual , normal skin E . Zinc .
turgor , moist oral mucosa .
B . Oliguria , no tears with
crying , less active than usual , * Regarding UTI in children , one of
sticky oral mucosa , normal or the following pathogens predisposes
slightly diminished skin turgor . to the formation of phosphate stones
C . Oliguria , no tears with by splitting urea to ammonia in the
crying , soft doughy skin . urine ?
D . Oliguria , sunken eye's , A . Escherichia coli .
tenting , tachycardia , B . Pseudomonas .
hypotension . C . Streptococcus faecalis .
E . Oliguria , hypotension . D . Proteus mirabilis .
E . Staphylococcus .
* Calculate the first hour fluid
management for a 25 kg 6 year old * The CBC for one year old showed
child with severe dehydration loss microcytic hypochromic anemia . All
10% of body weight ? of the following are correct
A . 150 ml differential diagnosis except ?
B . 180 ml A . IDA .
C . 200 ml B . Blackfan Diamond anemia .
D . 220 ml C . Thalassemia .
E . 250 ml D . Sideroblastic anemia .
E . Lead poisoning .
* All of the following can occur in
patients with Kawasaki disease * The most common cause of childhood
except ? meningitis in Jordan is ?
A . Coronary artery aneurysm . A . Streptococcus pneumonia .
B . Aseptic meningitis . B . Staphylococcus aureus .
C . Aseptic sterile pyuria . C . Mycobacterium tuberculosis .
D . Hydrops of the gallbladder . D . E coli .
E . Osteomyelitis . E . Neisseria meningitis .

* Blood that is used for transfusion * The CSF of 3 year old child with
purposes routinely screened for suspected meningitis showed : WBC
serologic markers for all of the 150 , Neutrophils 10% , Protein 50
following disease except ? mg/dl and CSF glucose 90 while blood
A . HBV glucose 100 . His CSF is most
B . HSV consistent with ?
C . Syphilis A . Bacterial meningitis .
D . CMV B . Fungal meningitis .
E . AIDS C . Tuberculous meningitis .
D . Early viral meningitis .
* All of the following about BCG E . Normal CSF analysis for age
vaccine are true except ?

‫اسئلــة االطفـــال‬ ‫ اسامه الخزاعله‬: ‫الدكتور‬


Page 7
* All of the following are D . Stage 4B Hodgkin lymphoma .
recognized as normal findings in a E . Stage 2A Hodgkin lymphoma
newborn baby except ?
A . Erythema toxicum on trunk . * In DKA all are true except ?
B . Lanugo hair on extremities . A . Insulin should not be given IV
C . Vaginal blood spotting . bolus .
D . Mongolian blue spot on low B . PH is below 7.3 and / or
spinal area . bicarbonate is less than 15 mg/L .
E . Group of vesicles in close C . Positive ketones bodies in serum
proximity on scalp . and urine .
D . Nacl solution should be given
* The most common type of cerebral initially .
palsy ? E . Nahco3 is given after 2 hours of
A . Atonic . starting treatment .
B . Ataxic .
C . Spastic . * One year old boy brought to
D . Hypotonic . emergency department for an attack
of generalized tonic seizures . In
* The therapy that can remove the the management of his seizure all
potassium from the body is ? are true except ?
A . 10 % calcium gluconate . A . Serum glucose , Ca , Na . Should
B . Bicarbonate Nahco3 . be measured .
C . Nebulized albuterol . B . First line of treatment with
D . Insulin . E . Kayexalate . antiepileptic drugs is
* The initial investigation for benzodiazepine .
short stature includes all except ? C . Phenytoin is given IV over 20
A . CBC minutes .
B . TFT D . When the cause is hypoglycemia 2
C . Growth hormone cc /kg of dextrose 10% is given IV .
D . KFT E . Calcium gluconate is given
E . Bone age intramuscular when it's serum level
is low .
* 11 month old girl had an illness
characterized by fever up to 39.4 * Neonatal seizures may present by
for 3 days , then fever subsided but all of the following picture
followed by a small pink macules except ?
over the trunk . The most likely A . Jitteriness .
diagnosis is ? B . Sucking .
A . Pityriasis rosea . C . Apnoea .
B . Measles . D . Nystagmus .
C . Scarlet fever . E . Eye blinking .
D . Roseola infantum .
E . Erythema infectiosum . * In an inherited disease , the
male's and female's are equally
* One of the following is not affected . The risk of siblings
considered as Jones criteria for having the disease is 25 % if both
rheumatic fever ? parents are carriers . The disease
A . Carditis . is inherited as ?
B . Arthralgia . A . Autosomal recessive .
C . Erythema marginatum . B . Autosomal dominant .
D . Sydenham Chorea . C . Mitochondrial .
E . Short PR interval . D . X linked recessive .
E . X linked dominant .
* Involvement of lymph node regions
on both sides of the diaphragm , may * A 5 day old full term newborn
include spleen with fever , night presented with neonatal jaundice
sweats , weight loss is called ? after the second day . His total
A . Stage 1A Hodgkin lymphoma . bilirubin is 20 mg/dl , direct
B . Stage 2B Hodgkin lymphoma . bilirubin is 0.5 mg/dl .
C . Stage 3B Hodgkin lymphoma .

‫اسئلــة االطفـــال‬ ‫ اسامه الخزاعله‬: ‫الدكتور‬


Page 8
His weight is 3.3 kg . Clinically he * All of the following parameters
looks well with no abnormal finding are used to monitor infant baby with
apart from jaundice . The management Hypovolemic shock except ?
of this case is by ? A . Level of consciousness .
A . Cessation of breast milk . B . Capillary refill time .
B . Phototherapy . C . Pupillary reaction .
C . IV fluid and antibiotics . D . Blood pressure .
D . Blood exchange . E . Skin turgor .
E . Dextrose 10% orally at home and
follow up in 24 hours * Regarding idiopathic
thrombocytopenic purpura all of the
* All of the following associations following are true except ?
regarding genetic disorders are true A . It is the most common cause of
Except ? acute onest of Thrombocytopenia in
A . Down syndrome and atlantoaxial otherwise healthy child .
subluxation . B . The peak age is 1 _ 4 years of
B . Turner syndrome and aortic valve age .
anomaly . C . Usually preceded by viral
C . Alagille syndrome and infection 1 _ 4 weeks before the
Cholestatic jaundice . onest .
D . Klinefelter syndrome and short D . Platelets size is normal or
stature . increased .
E . Prader willi syndrome and E . Spleen is enlarged in majority
obesity . of the case's .

* All of the following are live * A 5 months old infant brought by


attenuated vaccines except ? his mother who said that he had
A . Oral polio vaccine . history of vomiting 3 _ 4 times
B . Rotavirus vaccine . daily . His birth weight is 2.6 kg .
C . Measles vaccine . He looks well with no abnormal
D . Varicella vaccine . clinical findings . He weight
E . Hepatitis A vaccine currently is 5.8 kg . The best
management for this case is ?
* In first rescuer basic life A . Omeprazole .
support a 7 years old child with B . Ranitidine .
unwitnessed cardiac arrest . C . Metoclopramide ( Plasil ) .
Activation of the emergency response D . Domperidone ( Motilium ) .
system occurs ........ , E . No treatment is needed .
and the compression to ventilation
ration is ...... ? * 3 months old infant presented with
A . Initially while getting the lethargy , irritability ,
AED , 30:2 . tachypnea , reduced movements ,
B . Initially while getting the decreased body temperature . The
AED , 15:2 . best next step is ?
C . After 2 minutes of CPR , 30:2 . A . To give him Ceftriaxone at
D . After 2 cycles of CPR , 30:2 . home .
E . After 2 minutes of CPR , 15:2 . B . To do septic workup and send him
home for the next day .
C . Admission to the hospital .
* In a 5 year old child with D . Antipyretic and Ampicillin at
osteomyelitis , the most common home .
causative organism is ? E . To do septic workup and give him
A . Staphylococcus aureus . Ceftriaxone and send him for the
B . Escherichia coli . next day home .
C . Group B streptococcus .
D . Salmonella typhi .
E . Haemophilus influenzae type B

‫اسئلــة االطفـــال‬ ‫ اسامه الخزاعله‬: ‫الدكتور‬


Page 9
* Five year old male , known case of
* You deliver an infant who has frequently relapsing nephrotic
shoulder dystocia and at 1 minute it syndrome on steroid treatment . This
does not cry , as well as has flexed patient is at risk for all of the
extremities , irregular following except ?
respiration , a bluish A . Peritonitis .
color , heart rate 90 BPM . Most B . Malnutrition .
appropriate Apgar score for this C . Bleeding tendency .
infant ? D . Hyperlipidemia .
A . 1 E . Immunosuppression .
B . 2
C . 3
D . 5 * The leading cause of death among
E . 6 school children is ?
3* What is the approximate caloric A . Congenital anomalies .
need of a normal full term infant B . Injuries .
through the first year of life A . C . Malignant disease .
25 kcal / kg / day . D . Communicable disease .
B . 50 kcal / kg / day . E . Respiratory illnesses .
C . 75 kcal / kg / day .
D . 100 kcal /kg/day . * One week old newborn presented
E . 125 kcal / kg /day . with Cholestatic jaundice ,
hypoglycemia and seizure . You are
* The most common hereditary expecting Galactosemia as a
bleeding disorder is ? diagnosis . The gold diagnostic test
A . Hemophilia A . is ?
B . Hemophilia B . A . Plasma galactose level .
C . VWD . B . Liver biopsy .
D . DIC . C . Ophthalmology examination for
E . ITP . cataract .
D . Urine for reducing substance .
* Regarding congenital E . Measurement of GALT ( galactose
hypothyroidism one is true ? _1_ phosphate uridyltransferase
A . Most of the case's are secondary enzyme ) activity in Erythrocyte .
.
B . All case's must be treated with * One of the following disease can
T4 . cause aplastic anemia in a child
C . Treatment is usually stopped with hemoglobinopathy ?
after 8 months . A . Roseola infantum .
D . Most of the case's are B . Erythema infectiosum .
familial . C . Measles .
E . Symptoms are often seen by the D . Hand foot mouth disease .
end of the first week of life . E. Chickenpox .

* Regarding immune deficiency, all * Charcoal is ineffective in all of


of the following associations are the following poisoning except ?
true except ? A . Corrosive agents .
A . Hypocalcemia and DiGeorge B . Kerosin .
syndrome . C . Iron .
B . Delayed umbilical cord D . Lead .
detachment and leukocyte adhesion E . Aminophylline .
defect .
C . Recurrent Neisseria meningitis * Central precocious puberty in
and hyper IgM syndrome . boy's starts with increased
D . Severe progressive infectious testicular volume before the age ?
mononucleosis and X- linked syndrome A . 7 years .
. B . 8 years .
E . Recurrent cutaneous abscesses C . 9 years .
and hyper lgE syndrome . D . 10 years .
E . 6 years .

‫اسئلــة االطفـــال‬ ‫ اسامه الخزاعله‬: ‫الدكتور‬


Page 10
3- I.V.F
* Four weeks old male infant 4- I.V corticosteroids
presented with tiredness and rapid 5- NSAID
breathing during feeding . He has to
stop sucking frequent to catch his * All of the following are
breath . Which of the following is characteristic respiratory
the most appropriate investigation ? complications or features of
A . CBC test . fibrosis in children except:
B . ABG test . 1- Recurrent bronchiolitis
C . Echocardiography . 2- Pan sinusitis
D . Electroencephalography . 3- Bronchiectasis
E . Serum electrolytes . 4- Nasal polyposis
5- Pulmonary embolism

* All of the following congenital


heart defects causes excessive
pulmoray blood flow except:
* All are true about fetal 1- VSD
circulation except ? 2- ASD
A . The oxygenated blood is 3- Tertalogy of falot
transferred from the mother to the 4- PDA
fetus through the umbilical vein . 5- AVSD
B . The Foramen ovale is the Foramen
between two atria . * All of the following can causes
C . The pulmonary vascular hematuria in children except :
resistance is higher in the neonate 1- UTI
than in a fetus . 2- G6PD deficiency
D . The umbilical cord consists of 2 3- Trauma
arteries and 1 vein . 4- Renal stone
E . The shunt in the fetus is a 5- Schistosomiasis
right to left shunt .
* The most common cause of
* All are causes of overwreight gastroenteritis in children is:
except:‫الكل صحيح‬ 1- Rota virus
1- Psuedohypoparathyroidism 2- E.coli
2- Pradder Will syndrome 3- Food poisoning
3- Down syndrome
4- Turner syndrome * The most common organism causing
5- Adrenal insufficiency acute otitis media is :
1- Strep. Pneumonia
* All are X-linked diseases except: 2- Heamophilus influenza
1- Hemophilla A 3- Group A streptococcus
2- G6PD deficiency 4- Adenovirus
3- Duchenne muscular dystrophy 5- Pseudomonas aeruginosa
4- Hypophosphatemic rickets
5- Hereditary spherocytosis * The usual manifestations of
central precocious puberty in girls
* All the following lab findings are include :
true regarding iron deficiency 1- Increased bone age,LH and
anemia : estradiol levels
1- Low ferritin 2- Increased bone age and estradiol
2- Low platelets with decreased LH
3- Low saturation 3- Increased LH,FSH, and
4- Inceased TIBC testosterone with decreased
5- Decreased MCV estradiol
4- Delayed bone age and increased
* ALL of the following can be given LH,FSH AND estradiol
in treatment of anaphylaxis except: 5- Normal bone age with
1- Oxgen dereasedLH,prolactin and esrtadiol
2- Epinephrine

‫اسئلــة االطفـــال‬ ‫ اسامه الخزاعله‬: ‫الدكتور‬


Page 11
* The most common cause of * The objectives of vaccination
hereditary bleeding disorder is : program are all the following except
1- Hemophilia A :
2- Hemophilia B 1- To minimize mortality and
3- Von Willebrand disease morbidity
4- Factor 7 deficiency 2- To vaccinate all pregnant women
5- Factor 10 deficiency at the child bearing age with
tetanus toxiod
* Treatment of acute 3- To ensure that all children(0-*
laryngotracheobronchitis (croup) may years of age are protected against
all except: of childhood diseases
1- Oral or I.M dexamethasone 4- To provide vaccination for small
2- Oral prednisolone pox
3- Nebulizer Racemic epinephrine 5- To promote the growth and
4- Cool steam development of new generation of
5- Anti- histamine children

* The number of deciduous teeth is * All of the following is true about


1- 24 strepococcus pyogenes pharyngitis
2- 20 except:
3- 22 1- Purulant exudate and peterchial
4- 18 spots on soft palate can be found on
5- 32 examination
2- Penicillin is the best drug of
choice for treatment
3- Antibiotic therapy is recommended
* The first sign of puberty in girls to protect against post
is streptococcus glumerulonephritis
1- Onset of menstruation 4- Fine papular (sand-paper) rash
2- Pubic hair can be found
3- Axillary hair 5- Cervical lymphadenopathy is
4- Appearance of breast buds common
5- Growth spurt

* The most common organism causes


pleural effusion is :
1- Staph. Aureus
2- Strep . pneumonia
3- Group A streptococcus * 2 month old child , presented to
4- Chlamydia trachomatis the pediatrician with tongue
5- E.coli Fasciculation and generalized
Hypotonia , feeding difficulty and
* The most common cause of central absence of deep tendon reflexes .
precocious puberty in girls is : The most likely diagnosis is ?
1- pitutary adenoma A .Guillain barre syndrome
2- ovarian cysts B .Duchenne muscular dystrophy
3- intracranial hemorrhage C .Spinal muscular atrophy
4- idiopathic D .Tuberous sclerosis
5- CNS tumors E .Lesch _ Nyhan disease

* The most common cause of ambiguous * Vitamin D requirement for infants


genitalia in girls is : in physiological state is ?
1- 21 hydroxylase deficiency A . 200 I.U
2- 11 hydroxylase deficiency B . 400 I.U
3- Turner syndrome C . 600 I.U
4- Maternal exposure to androgen D . 4000 I.U
5- Androgen insensitivity syndrome E . 800 I.U

* Newborn presented to the


pediatrician with main complain of

‫اسئلــة االطفـــال‬ ‫ اسامه الخزاعله‬: ‫الدكتور‬


Page 12
bleeding from umbilical scar . D . Hemophilia A
History of baby delivery at home , E . Osteogenesis imperfecta
which of the following is the most
common cause of these signs and * Preterm baby is ?
symptom ? A . Weight less than 2.5 kg
A . Bleeding diathesis regardless of gestational age .
B . ITP B . Above completed 37 weeks
C . Hemolytic disease of the gestational age .
newborns C . Below completely 37 gestational
D . Vitamin C deficiency age
E . Vitamin K deficiency D . Below completed 40 weeks
gestational age .
* Treatment of choice for mastitis E . Immature lungs regardless of
neonatorum is gestational age.
A . Reassurance of the parents
B . Antibiotics topically * Infant of diabetic mother has all
C . Warm compression of the following except ?
D . Massage therapy A . Hypomagnesemia
E . Bromocriptine therapy B . IUGR
C . Hyperbilirubinemia
* All of the following are seen in D . Respiratory distress syndrome
Down syndrome except ? E . Hyperviscosity
A . Brushfield spots
B . Hypotonia * Newborn baby present with Cyanosis
C . Mongolian spots immediately after birth , the most
D . Atlantoaxial instability common cause is ?
E . Hypothyroidism A . TOF
B . TGA
* Most common cause of mental C . TAPVR
retardation in children is ? D . Tricuspid atresia
A . Sociocultural factors E . Truncus arteriosus
B . Psychological factors
C . ADHD * The most important risk factor for
D . Chromosomal factors Necrotizing enterocolitis is ?
E . Economical factors A . Prematurity
B . Postmaturity
C . Infant of diabetic mother
D . Congenital hypothyroidism
E . Black race

* Newborn baby is presented in


second day postoperative delivery
* Screening program in Jordan with chief complain of repeated
include ? vomiting . On abdominal film present
A . Congenital hypothyroidism , G6PD Double bubble sign , the most common
, hearing loss cause is
B . G6PD , Phenylketonuria , hearing A . Esophageal atresia
loss . B . Duodenal atresia
C . Congenital hypothyroidism , C . Jujenal atresia
Phenylketonuria , hearing loss . D . Congenital Pyloric stenosis
D . G6PD , Phenylketonuria , E . Imperforate anus
congenital hypothyroidism . * Newborn baby is presented with
E . Congenital hypothyroidism , jaundice , laboratory results show
Phenylketonuria . umbilical cord blood Hb 12.5 mg/dl ,
total serum bilirubin is 8 mg/dl.
* X_linked recessive inheritance The best next step in management
include ? is ?
A . Hereditary spherocytosis A . Phototherapy
B . VWD B . Reassurance of the parents that
C . Chorea Huntington is physiological

‫اسئلــة االطفـــال‬ ‫ اسامه الخزاعله‬: ‫الدكتور‬


Page 13
C . Immediately blood transfusion C . Give him antibiotics and go home
operation D . ICU admission and ENT
D . Admission and repeat testing consultation
after one hour .

* One of the following is true about


congenital hypothyroidism ?
A . Mental retardation is a rare * All of the followings
complication immunodeficiencies disorders are
B . If untreated , child will be associated with malignancy except
suffer from hypertonia A . Ataxia telangiectasia
C . It is mostly asymptomatic at B . Selective IgA deficiency
birth C . Chediak Higashi syndrome
D . Presented with microglossia D . Wiskott Aldrich syndrome
E . T3 , T4 , TSH are low in serum E . Hyperimmunoglobulin M syndrome

* One is true about hereditary * One of the following is live


spherocytosis ? attenuated vaccine ?
A . History of father that has Diphtheria
gallbladder stones and Splenectomy Hepatitis B
B . It is X_linked recessive Tetanus
C . MCH is low Hib
D . Diagnosis is confirmed by blood Rubella
smear
E . Folate is not beneficial * About snake bite injury , all are
true except ?
* All of the following are true A . Cold ice is used
about neonatal sepsis except ? B . Tourniquet using is necessary
A . Prematurity is a risk factor C . Incision and suction of the bite
B . Signs and symptoms are non is true
specific D . Antivenin administration if
C. Present with hypothermia and available
hyperthermia E . Compression bandage and
D . Most common microorganism is E immobilization are true
coli
E . If meningitis present , so treat * Causes of neonatal seizures ,
with Ampicillin and cefotaxime include all of the following
except ?
* 3 months old infant is presented Tuberous sclerosis
with parents to emergency room with Hypomagnesemia
chief complain hypoactivity , Hypocalcemia
fever , the best next step in Hypoglycemia
management is ? Febrile convulsions
A . Admission and sepsis work
B . Admission and symptomatic * All of the following statements
treatment are true except ?
C . Admission and observation A .Vitamin k deficiency causes
D . Chest X_Ray , if positive give bleeding tendency
him antibiotics B . Vitamin D deficiency causes
E . Lumbar puncture , if positive Rachitis
give him antibiotics C.Vitamin A deficiency causes
craniotabes
* Child 6 years old , is presented D . Vitamin B6 deficiency causes
with clinical picture of acute neonatal seizures
Epiglotitis , the best next step in E . Vitamin C deficiency causes Gum
management ? bleeding
A . Give him antibiotics and
admission to floor beds * All are true about Klinefelter
B . O2 therapy and admission to syndrome except ?
floor beds A . Karyotype 47 , XYY

‫اسئلــة االطفـــال‬ ‫ اسامه الخزاعله‬: ‫الدكتور‬


Page 14
B . Tall stature A . It is characterised by edema
C . Microorchidism B . Hypoproteinemia
D . Mental retardation C . Mainly it is due to inadequate
E . Decreased upper : lower segment calories
ratio D . Heart failure is one of the
complication
* Head circumference in 1 year old E . Occasionally Hepatomegaly
infant ?
40_42 cm * A 7 years old boy has recently
42_44 cm problems with bedwetting , on
44_46 cm physical examination appear that boy
46_48 cm has Enuresis and Hyperventilation
48_50 cm syndrome , this boy is expected to
have all of the following except ?
A . Serum PH < 7.4
B . PCO2 < 35 mmHg
C . Normal anion gap
D . Ketones bodies in serum and
urine
* Alarming sign of development , E . Autoantibodies against cells of
include all of the following langerhans
except ?
A . Urine control in day time in 1.5 * A 8 years old boy is presented
years old with parents with chief complain of
B . Walking well in 1.5 years old Enuresis , all of the following
C . Know name , sex , age at 3 years investigations should be done except
old ?
D . Pincer grasp in 9 months A . Urine culture .
E . Dominance of the hand in 1 years B . Urine analysis .
old C . Blood glucose .
D . IVP
* Newborn baby , at birth weight was E . Ultrasound abdomen and pelvis .
3.5 kg , at one year old will be * First vaccine that will be damaged
weighted ? by heating is
9.5 kg OPV
10 kg IPV
11 kg BCG
10.5 kg DTap
11.5 kg Hib

* Side effects of long-term steroid * All of the following vaccines


therapy , include all of the affected by freezing except ?
following except ? A . Diphtheria
PUD B . Pertussis
Eosinophilia C . Measles
Osteoporosis D . Tetanus
Skin striae E . IPV
Weight gain
* All of the following are true
* Physiological infants requirements about hepatitis A except ?
, include all of the following are A . This is very contagious
true except ? disease .
A . Vitamin A 4000 I.u / day . B . This is preventable disease by
B . Iron 10 mg / day vaccination .
C . Calories 110 kcal / kg / day C . May be represent nosocomial
D . Vitamin D 1200 I.u / day infection .
E . Vitamin C 50 mg / day D . In children it is more dangerous
than in adults
* All of the following are true
about Kwashiorkor , except ?

‫اسئلــة االطفـــال‬ ‫ اسامه الخزاعله‬: ‫الدكتور‬


Page 15
E . If it occur in immunodeficiency * About celiac disease , all of the
baby , it will lead to fluminant following are true except ?
hepatitis and up to death A . Diarrhea
B . Failure to thrive
* About hepatitis B vaccine , one is C . Permanent intolerance to gluten
true ? D . Vomiting
A . It is live attenuated E . Increasing of appetite
B . It is safe in pregnancy
C . The total number of vaccine is 4 * All of the following are used in
D . It is not present in Jordan treatment of Acute attack of
vaccine program bronchial asthma except ?
E . The first dose is given in 1 A . Theophylline
years old B . Cromoglycate Na
C . Ipratropium bromide
* All of the following are true D . Corticosteroids
about Jordan vaccine program E . Magnesium sulfate
except ?
A . The total of Hep vaccine is * 5 years old child , presented with
three , given in 2 , 3 ,4 months of chief complain of recurrent attack
age of dry cough , at night mainly . On
B . MMR vaccine is given in 1 year physical exam wheezy chest , which
old of the
C . Measles vaccine is given in 6 following is most likely diagnosis ?
month of age Bronchial asthma
D . The first vaccine against Pneumonia
poliomyelitis is IPV Bronchitis
E . This program not include Bronchiolitis
rotavirus vaccine Cystic fibrosis

* All of the following causes * All of the following are signs and
maculopapular rash except ? symptoms of severe attack of
Varicella bronchial asthma except ?
Measles PaCO2 normal
Fifth disease Bradycardia
Hand foot mouth disease Wheezy chest
Rubella PaO2 < 60 mmHg
Cyanosis
* A 5 month old infant present with
upper respiratory tract infection * One is true about otitis media ?
and wheezes , lab results increasing A . Tobacco smoking is one of the
lymphocytes ( lymphocytosis ) . risk factors
Which of the following is the most B . Mostly in children more than 10
likely diagnosis ? years old
Pneumonia C . In females more than males
Bronchitis D . Moraxella catarrhalis is the
Influenza most common cause
Bronchiolitis E . The drug of choice is
* 10 years old child , present with Ceftriaxone
chief complain of fever 38.5 , dry
cough , on chest X_Ray bilateral
patchy infiltration , CBC appear
normal , child do not appear ill ,
which of the following the most
likely diagnosis ?
Streptococcus pneumonia * 12 years old female present with
Viral pneumonia parents , with chief complain of
Bronchitis pain suprapubic , on urine culture
Mycoplasma pneumonia is positive for Pseudomonas
Bronchiolitis aeruginosa , which of the following
is the best for treatment ?

‫اسئلــة االطفـــال‬ ‫ اسامه الخزاعله‬: ‫الدكتور‬


Page 16
Amoxicillin E . If history of food poisoning ,
Clarithromycin the cause mostly Staphylococcus
Rifampicin aureus
Ceftazidime
Streptomycin

* The following ABG , PH = 7.2 ,


PCO2 = 65 mmHg , HCO3 = 25 mg/dl , * All of the following are true
which of the following is the most about Pyloric stenosis except ?
likely diagnosis ? A . Polyphagia
A . Uncompensated metabolic acidosis B . Hypochloremia
. C . Ultrasound is essential for
B . Uncompensated respiratory diagnosis
acidosis D . Bile stained vomitus
C . Mixed metabolic and respiratory E . Pylorotomy is the treatment of
acidosis . choice after electrolytes correction
D . Compensated respiratory acidosis .
.
E . Uncompensated respiratory * Most common cause of recurrent
alkalosis . abdominal pain in children is ?
Meckels diverticulum
* All of the following are true UTI
about ASD except ? Intussusception
A . Loud first heart sound Celiac disease
B . Right axis deviation on ECG Gastroenteritis
C . Cyanosis
D . Pulmonary blood flow increased * A 5 years old boy is seen with
E . No need for infective maculopapular lesions on the legs
endocarditis prophylaxis and buttocks , he is complain of
abdominal pain , in urine analysis
* All of the following are signs and present
symptoms of heart failure in Hematuria , other lab findings are
children except ? within normal , the most likely
A . Feeding difficulty diagnosis ?
B . Sweating during feeding ITP
C . Failure to thrive TTP
D . Orthopnea HSP
E . Cyanosis SLE
Kawasaki disease
* All of the following are cyanotic
heart disease except ? * 4 years old boy suffers from
TOF developmental delay . Lab findings
Tricuspid atresia are Ca 7.1 mg/dl , PO4 2.1 mg/dl ,
Coarctation of aorta ALP 1800 I.u . Which of the
Ebstein anomaly following is the most likely
TGA diagnosis ?
Nutritional rickets
* All of the following are true Hypoparathyroidism
about gastroenteritis in children Pseudo_hypoparathyroidism
except ? Chronic kidney disease
A . All Salmonella induced Kwashiorkor
gastroenteritis need antibiotics
B . Most common cause of Acute * All of the following are true
diarrhea in children is about DM type two except ?
gastroenteritis . A . Mostly in old age , but recently
C . Rotavirus is the most common start to appear in young age .
cause in infancy . B . Associated with obesity .
D . If positive history of C . DKA does not occur
antibiotic use , the cause will be C D . Lifestyle modification is
. difficile helpful .

‫اسئلــة االطفـــال‬ ‫ اسامه الخزاعله‬: ‫الدكتور‬


Page 17
E . In some cases insulin is used * Most common cancer in children
for treatment is ?
CML
* All of the following are true ALL
about DM type one except ? Brain tumor
A . It is autoimmune Lymphoma
B . Mostly in children and young Neuroblastoma
ages
C . 20_25% diagnosed with DKA v * All of the following are causes of
D . Acanthosis nigricans prenatal Polyhydramnios except ?
E . Insulin is essential for Potor syndrome
treatment Anencephaly
Duodenal atresia
Diabetic mother
Diaphragmatic hernia

* All are true suggest the diagnosis


* All of the following are true of febrile seizure except ?
about IDA except ? A . 6 months _ 6 years old
A . Decreased serum Fe . B . Generalized tonic clonic
B . Decreased hemosiderin . C . Rapid increase of temperature
C . Increased RDW . D . Family history of epilepsy
D . Decreased transferrine E . No EEG or neuroimaging
saturation . findings .
E . Decreased TIBS

* About acrodermatitis enteropathica * All are true about


, all are true except ? Neurofibromatosis type one except ?
A . Associated with congenital A . Cafe au lait spots
anomaly . B . Bilateral acoustic neuroma
B . Associated with zinc deficiency C . Freckling in axilla
C . Perioral skin rash D . Autosomal dominant inheritance
D . Diarrhea E . Optic gliomas
E . Alopecia
* All are true about Guillain barre
* 10 years old child complain of syndrome except ?
easy bruising , bleeding on knee A . Ascending Demyelinating
joints , petechiae on skin of upper polyneuropathy .
extremities , lab findings : INR B . Post infection mostly
normal , PT normal , PTT increased , Campylobacter jejuni
bleeding time increased , platelets C . Abnormal light _ pupil reaction
normal count and function . Which of D . Autoimmune instability
the following is the most likely E . Protino_cellular dissociation on
diagnosis ? CSF analysis
DIC
Vitamin k deficiency * Tuberculin mantoux skin test is
ITP considered positive if ?
VWD A . > 10 mm induration ‫لم يأخذ المطعوم‬
HSP B . > 10 mm Erythema
C . > 5 mm induration
* The treatment of choice of G6PD D . > 15 mm Erythema
deficiency is ? E . > 15 mm induration ‫اخذ المطعوم‬
A . Splenectomy
B . Supportive and blood transfusion * Increasing of alpha fetoprotien is
if needed noted in ?
C . Bone marrow transplantation . ALL
D . Plasmapheresis . Lymphoma
E . No need for treatment . Pheochromocytoma
Hepatoblastoma
HCC

‫اسئلــة االطفـــال‬ ‫ اسامه الخزاعله‬: ‫الدكتور‬


Page 18
Pediatrics 7,2016 Meconium aspiration syndrome

* All are true matching about * All are true about Caput
antidote and substrates except ? succedaneum except ?
A . Ethylene glycol _ Fomepizole . A . Diffuse edema on the scalp .
B . Iron _ Desferrioxamine . B . Don't cross suture line .
C . INH _ Thiamine . C . Most common cause vacuum vaginal
D . Lead _ DMSA . delivery
E . Sulfonamides _ Methylene blue . D . Rapidly resolves .
E . No anemia or jaundice .
* All are signs and symptoms of
severe dehydration except ? * All are true physiological
A . 15 % infant weight loss jaundice except ?
B . Anuria A . Start after 24 hours .
C . Tachycardia B . Peak 3_4 th day of life .
D . Pulging fontanel C . Resolves by 21 days .
E . Comatose D . Max 14 mg/dl
E . Indirect bilirubin elevation .
* All are causes of hypoglycemia
except ? * Most common cause of neonatal
Hypothyroidism seizures is ?
CAH Brain injury
Galactosemia Idiopathic
Gigantism Meningitis
Reye syndrome Hypoglycemia
Kernicterus

* All are causes of floppy infant


except ?
Sepsis
* All are true criteria of DKA Post_term
except ? Hypoglycemia
A . Hyperglycemia > 11 mmol/l Trisomy 21
B . Anion gap 12 Werdnig Hoffman disease
C . PH < 7.3
D . Bicarbonate < 15 mmol/L * Self resolving bluish mouth floor
E . Serum ketones swelling is called ?
Epsteins pearls
* All are causes of high anion gap Ranula
metabolic acidosis except ? Thrush
Aspirin Vernix
Uremia Myxoma
Lactate
Sepsis * All of the following are neural
Renal tubular acidosis tube defect except ?
A . Anencephaly
* All are complications of small for B . Encephalocele
gestational age except ? C . Spina bifida occulta
Hypothermia D . Myelo_meningocele
Hypoglycemia E . Dandy _ Walker malformation
Anemia
Necrotizing enterocolitis * Most common cause of heart failure
Meconium aspiration syndrome in children
Congenital heart defects
* All are association with Cardiomyopathy
prematurity except ? Myocarditis
Apnoea Kawasaki disease
Jaundice HTN
Retinopathy
PDA

‫اسئلــة االطفـــال‬ ‫ اسامه الخزاعله‬: ‫الدكتور‬


Page 19
* All of the following infection can * All are true about Croup except ?
cause Hydrops fetalis except ? A . It's viral
Toxoplasmosis Laryngotracheobronchitis .
Rubella B . Children age 6 months _ 6
CMV years .
Candidiasis C . High grade fever .
Parvovirus B19 D . Barking cough .
E . Hoarseness of voice
* All are clinical features of heart
failure in children except ? * All are causes of Acute diarrhea
Tachypnoea except ?
Failure to thrive Antibiotics
Hepatomegaly HSP
Splenomegaly HUS
Tachycardia Celiac disease
Intussusception
* All of the following are causes of
systolic murmur except ? * Most common cause of failure to
Coarctation of aorta thrive ?
Secundum ASD Malnutrition
HOCM Emotional deprivation
TOF DM
PDA Malabsorption
Congenital heart disease
* All are acyanotic congenital heart
disease except ? * All are true about celiac disease
PDA except ?
COA A . Associated with type one DM ,
Dextrocardia Down
TAPVR B . It's gluten protein
HOCM enteropathy .
C . Chronic diarrhea .
D . Increased appetite .
E . Peripheral edema .

* All are true about Jones criteria * All are causes of Hematuria except
of Rheumatic fever except ? ?
Polyarthritis Alport syndrome
Long PR interval HSP
Sydenham Chorea SCA
Erythema nodosum MCD
Elevated ESR/CRP TB

* All are common causes of Stridor


except ? * Rib_notching on CXR is
Choanal atresia characteristic for ?
Bronchitis HOCM
Macroglossia Aortic stenosis
Cystic hygroma PDA
Epiglotitis Coarctation of aorta
TOF
* All of the following are
bronchodilator for child with * All are causes of direct
bronchial asthma except ? hyperbilirubinemia except ?
Terbutaline Alpha one antitrypsin deficiency
Formoterol Wilson disease
Ipratropium Sepsis
Theophylline Thalassemia
Beclomethasone Toxoplasmosis

‫اسئلــة االطفـــال‬ ‫ اسامه الخزاعله‬: ‫الدكتور‬


Page 20
* All are causes of Polyuria A . Prader Willi syndrome
except ? B . Bardet Biedl syndrome
DI C . Addison disease
Fanconi syndrome D . Pseudo_hypoparathyroidism
Chronic renal failure E . Cushing disease
Hyperkalemia
Hyperglycemia * All are true about DM type one
except ?
* All are true about Pre_renal cause A . T_cell mediated B_cell
of Acute renal failure except ? destruction .
A . Urine osmolality < 350 mosmol/kg B . Anti GAD antibodies are positive
. .
B . Urine / plasma cr . ratio > 40 . C . Increased catabolism .
C . High urine specific gravity . D . Candida infection and skin
D . Urine Na < 20 mmol/l infection .
E . FE NA < 1% E . Associated with obesity .

* All are causes of constipation * All are signs and symptoms of


except ? hypoglycemia except ?
Hypothyroidism Anorexia
Opioids Sweating
Functional Syncope
Hirschsprung disease Tachycardia
Intussusception Tremor

* All are causes of indirect * All are causes of congenital


hyperbilirubinemia except ? hypothyroidism except ?
G6PD Thyroid dysgenesis
Leptospirosis Pendred syndrome
HUS Hashimoto thyroiditis
Gilbert syndrome Iodine deficiency
Crigler najjar syndrome Empty sella syndrome

* Most common cause of renal calculi * All are causes of Hypocalcemia


in children except ?
Congenital malformation A .Prematurity
Cystinuria B .Digeorge syndrome
Hyperurecemia C .Hypomagnesemia
Chronic UTI D .Pseudo _
Oxalosis Pseudo_hypoparathyroidism
E .Nutritional rickets
* All are causes of HTN in children
except ? * All are true about vitamin D
Polycystic kidney disease dependent rickets type one except ?
Renal artery stenosis A . Plasma calcium decreased .
Hypoaldosteronism B . Plasma PO4 decreased .
CAH C . Plasma alkaline phosphatase
Wilms tumor decreased .
D . 1.25(OH)2 D3 decreased .
* All are true about cystic fibrosis E . Plasma PTH increased .
except ?
A . Cough and wheezes . * All are diagnostic criteria for
B . Weight loss . SIADH except ?
C . Bloody diarrhea . A . Serum Na < 120 mmol/L .
D . Malabsorption . B . Plasma osmolality< 270 mosm/kg
E . Increased chloride level in C . Urine NA > 20 mmol/L
sweat . D . Normal renal , adrenal , thyroid
function .
* All of the following are causes of E . No Hypovolemia or hypervolemia .
obesity except

‫اسئلــة االطفـــال‬ ‫ اسامه الخزاعله‬: ‫الدكتور‬


Page 21
* All are signs and symptoms of 2 years old
congenital hypothyroidism except ? 1 year old
Prolonged time 15 months old
Obesity 9 months old
Constipation
Hypotonia
Dry skin * All are causes of macrocytosis
except ?
* All are causes of short stature A . Diamond Blackfan syndrome
except ? B . Fanconi anemia
Turner syndrome C . Myelodysplastic syndrome
Fragile x syndrome D . Thalassemia
Noonan syndrome E . Megaloblastic anemia
Down syndrome
Celiac disease * All are adverse prognostic factors
for ALL except ?
* All are true about congenital A . Age < 1 year old or > 10 years
adrenal hyperplasia except ? old .
A . Autosomal dominant inheritance . B . Philadelphia chromosome t (9:2*
B . 21 alpha hydroxylase deficiency C . Poor response to induction
is the most common form . agents .
C . Mutation in chromosome 6 In 21 D . Female gender
alpha hydroxylase deficiency type . E . High WBC at diagnosis .
D . Elevated plasma 17
hydroxy_progestrone level . * Koplik spots are pathognomonic for
E . Hyponatremia , Hyperkalemia , ?
metabolic acidosis . Scarlet fever
Measles
* Most common cause of gynecomastia Rubella
in children ? Parvovirus
Leydig cell tumor Human herpes virus 6
Sertoli cell tumor
Germ cell tumor * The triad of arthritis , colicky
Pubertal gynecomastia abdominal pain and palpable popular
Androgen insensitivity syndrome purpuric rash over buttocks and
lower legs for ?
* All are true about febrile Polyarteritis nodosa
seizures except ? Wegener Granulomatosis
A . Occur in infant and small Takayasu arteritis
children . HSP
B . Temperature > 39 Scleroderma
C . Vast majority of febrile
seizures are harmful . * All are causes of
D . 95_98% do not go onto develop Hyperpigmentation except ?
epilepsy . Peutz_Jeghers syndrome
E . Mostly generalized tonic Mongolian spot
clonic . Neurofibromatosis
Phenylketonuria
* Child can hop on one leg , go up Hypoadrenalism
and down stairs one leg at a time ,
ride bicycle at ? * Typical age for intussusception is
2 years old ?
2.5 years old A . 0_6 months
3 years old B . 6_18 months
4 years old C . 18_36 months
15 months old D . 36_48 months
E . 48_60 months
* Child can combine 2 words together
by the age * All are causes of Cholestatic
1.5 years old jaundice except ?

‫اسئلــة االطفـــال‬ ‫ اسامه الخزاعله‬: ‫الدكتور‬


Page 22
A . Caroli disease C . Erythema nodosum .
B . Cystic fibrosis D . Sydenham chorea .
C . Cholangitis E . Prolonged PR interval .
D . Budd Chiari syndrome
E . Biliary atresia * Which of the following diagnosis
is most likely in a child with HTN
* Which of the following is and past history of UTI ?
associated with good clinical A . Ureteropelvic junction
prognosis in acute lymph leukemia obstruction .
A . Being a female patient . B . Reflux nephropathy .
B . High WBC count . C . Wilms tumor .
C . Philadelphia chromosome ( t 9 : D . Renal artery stenosis .
2* . E . Posterior urethral valve .
D . Presenting before age of one
year . * All of the following are risk
E . T ( 4:1* translocation . factor for the development of
* One of the following is not a respiratory distress syndrome except
criteria for diagnosis of SLE ? ?
A . Malar rash . A . Prematurity .
B . Photosensitivity . B . Maternal diabetes .
C . Positive antinuclear C . Cesarean section .
antibodies . D . Acidosis .
D . Seizures and psychosis . E . Asphyxia .
E . Presence of WBC in urine more
than 50 / HPF .

* Which of the following serology * The therapy that can remove the
result that the individual received potassium from the body is ?
the HBV vaccine ? A . 10% Calcium gluconate .
A . Positive HBsAg , positive B . Bicarbonate .
HBsAb . C . Insulin .
B . Positive HBcAg , negative D . Kayexalate .
HBsAb . E . Nebulized albuterol .
C . Negative HBeAg .
D . Negative HBsAg , negative * The best test to evaluate an
HBsAb . asthma patient for degree of airway
E . None of the above . obstruction is ?
A . CXR .
* National immunization program in B . Peak flow meter .
Jordan cover all of the following C . Pulse oximeter .
disease except ? D . Methacholine challenge test .
A . Rubella . E . Spirometry .
B . Mumps .
C . Polio . * In sudden infant death syndrome
D . Pertussis . all are Predisposing factors
E . Hepatitis A . except ?
A . Prematurity .
* All are causes of Thrombocytopenia B . Smoker mother's .
except ? C . Positive position sleeping .
A . Leukemia . D . Positive family history .
B . HSP . E . Age more than 9 months .
C . Fanconi anemia .
D . Wiskott aldrich syndrome . * Breast milk is absolutely
E . Vancomycin . contraindicated in all of the
following except ?
* One of the following is not A . Mother on chemotherapy .
considered as Jones criteria for B . Mother who has HIV .
rheumatic fever ? C . Mastitis .
A . Carditis . D . Galactosemia .
B . Arthralgia . E . Primary lactase deficiency .

‫اسئلــة االطفـــال‬ ‫ اسامه الخزاعله‬: ‫الدكتور‬


Page 23
B . Cotrimoxazole .
* In a 5 years old child with C . Doxycycline .
osteomyelitis , the most common D . Rifampicin .
causative organism is ? E . Gentamicin .
A . Staphylococcus aureus .
B . Salmonella typhi . * Diabetes mellitus is diagnosed if
C . Haemophilus influenzae type B . fasting plasma glucose is more
D . Escherichia coli . than ?
E . Group A Streptococcus . A . 125 mg/dL .
B . 100 mg/dL .
* The most common cause of cerebral C . 140 mg/dL .
palsy is ? D . 200 mg/dL .
A . Neonatal Asphyxia . E . 110 mg/dL .
B . Epilepsy .
C . Meningitis . * The most common cause of
D . Head trauma . pericarditis is ?
E . Hydrocephalus . A . Streptococcus .
B . Staphylococcus .
* One of the following is not a C . Mycoplasma .
component of TOF D . Enteroviruses .
A . VSD . E . Tuberculosis .
B . P . S .
C . Overriding aorta . * Cutis marmorata occurs in ?
D . PDA . A . Down syndrome .
E . Right ventricular hypertrophy . B . Congenital hypothyroidism .
C . Sepsis .
D . Cold environment .
E . All of the above .

* Central precocious puberty in boys


starts with increased testicular
volume before the age ? * Which of the following is most
A . 7 years old . useful during the staging of Hodgkin
B . 8 years old . lymphoma ?
C . 9 years old . A . Clinical examination .
D . 10 years old . B . ESR level .
E . 6 years old . C . CT scan .
D . Karyotyping .
* All of the following can occur in E . Bone marrow biopsy .
patients with Kawasaki disease
except ? * One of the following seen in 21
A . Coronary artery aneurysm . hydroxylase deficiency ?
B . Osteomyelitis . A . High sodium .
C . Aseptic meningitis . B . Low testosterone .
D . Hydrops of the gallbladder . C . High cortisol .
E . Sterile pyuria . D . Low plasma renin activity .
E . High 17 hydroxyprogesterone .
* The following are considered to be
zoonotic disease except ? * 4 years old with sickle cell
A . Rabies . anemia is admitted to the hospital
B . Anthrax . because of shortness of breath ,
C . Hepatitis A . fever and chest pain . His pulse
D . Plague . oximeter was 80% . His Hb 8mg/dL .
E . Brucellosis . There was no organomegaly . His
symptoms are most likely secondary
* 10 years old child has Brucellosis to ?
. All of the following medications A . Sequestration crisis .
could be used to treat him except ? B . Acute chest syndrome .
A . Cefazolin . C . Aplastic crisis .

‫اسئلــة االطفـــال‬ ‫ اسامه الخزاعله‬: ‫الدكتور‬


Page 24
D . Vaso occlusive crisis .
E . Sepsis . * All of the following parameters
are used to monitor infant baby with
* All of the following are true Hypovolemic shock except ?
about Hemophilia except ? A . Level of consciousness .
A . Inheritance is x linked . B . Blood pressure .
B . Normal prothrombin time . C . Skin turgor .
C . Prolonged activated PTT D . Capillary refill time .
D . Treated by blood transfusion . E . Pupillary reaction .
E . Intramuscular injection should
be avoided . * All are used in acute exacerbation
of bronchial asthma except ?
* Which of the following is true A . Ipratropium bromide .
regarding stage 1A Hodgkin B . Steroids .
lymphoma ? C . Salbutamol .
A . Associated with increased LDH . D . Salmeterol .
B . May present with weight loss . E . Adrenaline .
C . It is best left untreated , so
called watch and wait . * All of the following can cause
D . It may be confined to the lymph early heart failure except ?
nodes on one side of the diaphragm . A . VSD .
E . Lymphadenopathy is usually B . ASD .
painful . C . PDA .
D . COA .
* A child presented with sever bowed E . Complete AV canal .
legs and flaring of both wrists ,
feeding history reveals he is * One of the following is a live
formula fed and receives a daily attenuated vaccine
supplement , A . Pertussis .
laboratory result demonstrate the B . Diphtheria vaccine .
following : High 25 ( oH ) 2D , low C . Mumps vaccine .
1.25 ( oH ) 2D , low calcium , D . Tetanus vaccine .
normal phosphorus . What diagnosis E . Injectable polio vaccine .
is most consistent with these
findings ? * Which of the following is the most
A . Cystic fibrosis . common cause of nappy rash ?
B . Hypophosphatemic rickets . A . Atopic eczema .
C . 22q11 deletion syndrome . B . Bacterial cellulitis .
D . Vitamin D refraction rickets C . Irritant dermatitis .
type . D . Candida infection .
E . Vitamin D deficiency rickets . E . Infantile seborrheic
* All of the following expected dermatitis .
initial workup for non febrile
convulsions except ?
A . Blood glucose .
B . Serum electrolytes . * A one year old child was brought
C . Serum ammonia . to the emergency room for vomiting
D . Brain CT scan . and diarrhea and dehydration . His
E . Electroencephalogram . sodium was 163 meq / L . Correction
of sodium and dehydration should be
* Regarding DDH ( Development done over ?
dysplasia of the Hip ) All are true A . 6 hours .
except ? B . 12 hours .
A . More common in boys . C . 24 hours .
B . Familial . D . 48 hours .
C . More common in first baby . E . 72 hours .
D . More common in breech
presentation . * The gold standard test to
E . The best test before 3 months of diagnosis Galactosemia is ?
age is by US A . Urine for substances .

‫اسئلــة االطفـــال‬ ‫ اسامه الخزاعله‬: ‫الدكتور‬


Page 25
B . Plasma galactose level . E . Staphylococcus .
C . Liver biopsy .
D.Measurement of GALT enzyme in * 15 month old presented with first
erythrocytes episode of wheezing . He has no
E . Ophthalmology examination for family history of atopy . The most
cataract . likely pathogen is ?
A . RSV .
* In assessing the development of an B . Rhinovirus .
infant , all of the following are C . Influenza .
red flags except ? D . Pertussis .
A . Fisting . E . Human metapneumovirus
B . Persistent posture .
C . Persistent primitive reflexes . * 4 months old infant should do all
D . Hand predominance after the age of the following except ?
18 month . A . Crawls .
E . Abdominal movement pattern . B . Coos .
C . Laughs and loud .
* Epidemic is ? D . Reaches and grasp object .
A . Constantly present . E . Support his when pulled to
B . Exhibiting seasonal pattern . sitting position
C . Occurring clearly in excess of
normal expectation . * The following is consistent with
D . Occurring clearly lower than Thalassemia minor ?
normal expectation . A . High MCV , high MCH , low RDW .
E . Global spread of new disease . B . Low MCV , low MCH , high RDW .
C . High MCV , low MCH , high RDW .
* Regarding west syndrome , all are D . Low MCV , low MCH , low RDW .
true except ? E . Low MCV , low MCH , normal RDW .
A . Presents in the first year of
life , peak onest 4 _ 7 months . * All of the following could cause
B . Presents as spasms associated indirect hyperbilirubinemia in a
with encephalopathy . newborn except ?
C . Hypsarrhythmia is the typical A . Physiological jaundice .
EEG finding B . Breast feeding jaundice .
D . Excellent prognosis , no risk of C . Cephalohematoma .
future seizures or developmental D . Crigler najjar .
delay . E . Cystic fibrosis .
E . Adrenocorticotropic hormone is
an effective treatment . * The formula daily need for an
infant who weight 5 kg is ?
* Regarding peptic ulcer disease in A . 750 ml .
children , all of the following are B . 500 ml .
true except ? C . 900 ml .
A . Duodenal ulcer are usually D . 1000 ml .
chronic . E . 350 ml .
B . NSAIDs medication causes gastric
ulcers . * 3 years old child can do all of
C . H pylori IgG titer correlates the following except ?
with the disease activity . A . Copies circle .
D . PPI are superior to H2 _ blocker B . Known his age .
. C . Knows his / her gender .
E . H pylori infection is D . Tells stories .
transmitted by Feco _ oral route E . Rides tricycle .

* Following pathogen is a phosphate


stone former in patient with UTI ? * Platelets disorder involved in the
A . E Coli . pathogenesis of bleeding in all of
B . Pseudomonas . the following diseases except
C . Streptococcus faecalis . A . ITP .
D . Proteus . B . VWD .

‫اسئلــة االطفـــال‬ ‫ اسامه الخزاعله‬: ‫الدكتور‬


Page 26
C . Hemophilia A . D . D/W 10% 10 cc bolus .
D . Bernard soulier syndrome . E . D/W 25% 10 cc bolus .
E . Glanzmann .

* All the following could be


prevented when you treat patient
with group A Streptococcus * All of the following association
( Streptococcus pyogenes ) regarding genetic disorders are true
pharyngitis except ? except ?
A . Peritonsillar abscess . A . Down syndrome and atlantoaxial
B . Rheumatic fever . subluxation .
C . Glomerulonephritis . B . Turner syndrome and aortic valve
D . Cervical lymphadenitis . anomalies .
E . Reactive arthritis . C . Alagille syndrome and
cholestatic jaundice .
* Which of the following drug D . Klinefelter syndrome and short
regimens is the most appropriate stature .
Chemoprophylaxis for adult household E . Prader willi syndrome and
contacts of a child with obesity .
meningococcal
meningitis ? * Which of the following types
A . Single dose ciprofloxacin . juvenile idiopathic arthritis is
B . Penicillin for 2 day's . most commonly associated with
C . IM Ceftriaxone . development of severe arthritis ?
D . Trimethoprim sulfamethoxazole A . Systemic onest juvenile
for 7 day's . idiopathic arthritis .
E . Penicillin and rifampin for 2 B . Pauciarticular juvenile
day's . idiopathic arthritis without ocular
involvement .
* In a 2 years old boy with C . Pauciarticular juvenile
recurrent wheezing , the following idiopathic arthritis with associated
is not considered risk factor to ocular involvement .
develop asthma ? D . Polyarticular rheumatoid factor
A . His mother has asthma . positive juvenile idiopathic
B . Had previous RSV infection . arthritis .
C . Has congenital heart disease . E . Polyarticular rheumatoid factor
D . Has eczema . negative juvenile idiopathic
E . Has allergic rhinitis . arthritis .

* Regarding Brucellosis , all are * Constitutional growth delay is


true except ? characterized by all of the
A . It's zoonotic disease . following except ?
B . Serology can be falsely positive A . Delayed pubertal growth spurt .
with vibrio cholerae . B . Bone age is equal to
C . Most patients will develop chronological age .
maculopapular rash . C . Height age 11 , bone age 11 .
D . The definitive diagnosis is by D . Delay secondary sexual
isolation of the organism by culture characteristics .
. E . Normal adult stature .
E . Endocarditis is a serious
complication . * You are expecting Guillain barre
syndrome in an 8 years old child .
* A 4 months old infant brought to All of the following physical
the emergency room with seizure . finding are expected except ?
His weight was 5 kg and serum A . Normal level of consciousness .
glucose 30 mg/dL . The best first B . Positive babinski .
line treatment of the following is ? C . Intact sensation .
A . Phenobarbital IV . D . Ascending motor weakness .
B . Phenytoin . E . Areflexia .
C . Diazepam IV or rectally .

‫اسئلــة االطفـــال‬ ‫ اسامه الخزاعله‬: ‫الدكتور‬


Page 27
* One of the following is false B . Serum glucose , urea and
regarding post Streptococcal electrolytes .
Glomerulonephritis in children ? C . Chest x Ray .
A . Typically presents with sudden D . Urine analysis .
onest of dark urine , edema and E . Vital signs .
HTN .
B . Commonly followers Streptococcal * A term newborn male , weighing 2.5
pharyngitis 2 _ 3 weeks from the kg at birth , is noted to have a
onest of infection . shortened ( hypoplastic ) left lower
C . Complete recovery occurs in most extremities , multiple scars on the
patients . trunk . Bilateral absence of the red
D . Early antibiotic treatment for reflex and microcephaly . During
Streptococcal infection will pregnancy which of the following
eliminate the risk of laboratory finding would likely have
Glomerulonephritis . been noted in the mother of this
E . Urine analysis usually shows newborn ?
RBCs , RBCs cast and mild A . Positive rubella IgM levels .
proteinuria . B . Positive varicella IgM levels .
C . Positive parvovirus IgM levels .
D . Positive CMV IgM levels .
E . Positive Toxoplasma gondii IgM
levels .

* A 3 day's old full term baby whose


weight is 2.5 kg was brought to your
* A healthy 20 month old girl found clinic for jaundice . His total
to have closed anterior fontanelle . bilirubin was 28 mg/dL , direct
Head circumference was normal . The bilirubin 2 mg/dL and Hb 13 g/dL .
most appropriate next step in the The best management for this newborn
evaluation of this child is ? is ?
A . Neurosurgery referral . A . Glucose water and phototherapy
B . Vitamin D supplement . at home .
C . Skull imaging . B . Stop breast feeding and assure
D . Referral to a pediatric the family .
neurology . C . Phototherapy .
E . This is a normal finding and no D . Blood exchange .
action is needed . E . Antibiotics .

* All of the following are true


about minimal change nephrotic
syndrome in children except ?
A . Usually don't progress to renal *An 11 month old girl had an illness
failure . characterized by fever up to 39.4
B . Often precipitated by and mild irritability for 3 day's .
respiratory infection . Fever then subsided but followed by
C . Steroids sensitive nephrotic small pink macules over trunk and
syndrome . extremities . The patient most
D . Complement C3 and calcium are likely has ?
low . A . Erythema infectiosum .
E . Age presentation is between one B . Pityriasis rosea .
and ten years . C . Roseola infantum .
D . Measles .
* 5 years old male presented with E . Scarlet fever .
abdominal pain , vomiting and
tachypnea . Clinical examination for * Regarding meningococcal meningitis
chest showed good air entry , one of the following is true ?
bilateral with no creeps or A . It is gram negative diplococci .
wheezes . All of the following B . Doxycycline is the recommended
investigation are important except ? prophylaxis for closed contacts .
A . Blood gases .

‫اسئلــة االطفـــال‬ ‫ اسامه الخزاعله‬: ‫الدكتور‬


Page 28
C . Duration of treatment is 2 to 3 A . 1000 cc of dextrose 5% in 1/4
weeks . normal saline per 24 hours .
D . Patients should be isolated till B . 1500 cc of dextrose 5% in 1/4
end of treatment . normal saline per 24 hours .
E . Mortality rate is higher than it C . 200 cc of normal saline over 20
is with meningococcemia minutes .
D . 50 cc of whole blood over 4
* 7 years old boy has abdominal pain hours .
and a rash that started several E . 200 cc of dextrose 10% over 20
day's ago . On examination you minutes .
notice a palpable purpuric rash over
his calves and buttocks with * An irritable 12 month old male has
swelling of both ankles . Abdominal a one week history of high fever and
exam is unremarkable . What is the macular truncal rash . Examination
most likely laboratory finding ? reveals bulbar conjunctivitis bright
A . Normal clotting parameters . red cracked lips , and cervical
B . Decreased bacterial count . adenopathy . What is the most
C . Low C3 complement levels . appropriate next step ?
D . Hypochromic microcytic anemia . A . Initiate airborne precautions .
E . Increase blood urea nitrogen and B . IV antibiotics .
creatinine C . IV corticosteroids .
D . IV gammaglobulin .
* 7 years old boy presented to your E . IV antibiotics for the infant
office after being exposed to his and oral antibiotics for all
grandmother who has tuberculosis household contacts .
disease . He is asymptomatic. His
PPD is 12 mm induration . His CXR * All of the following statements
was normal . Of the following the regarding the disappearance of
best next step in management of this primitive reflexes are true except ?
patient is ? A . Palmar grasp ref disappears by 2
A . Begin isoniazid preventive _ 3 months
therapy . B . Moro reflex disappears by 4 _ 6
B . Obtain early morning gastric months.
lavage culture 3 times . C . Parachute reflex disappears by 9
C . Reassurance and send home . _ 10.
D . Order an interferon gamma D . Tonic neck reflex disappears by
release assay . 6 _ 7 month
E . Start INH , Rifampicin , E . Rooting reflex disappears by 1 _
ethambutol and pyrazinamide 2 months 7

* Which of the following is


characteristic of kernicterus ?
A . Direct hyperbilirubinemia .
B . Opisthotonus is an early
manifestation .
C . Deafness is reversible .
D . Affect mainly the temporal
region .
E . Occurs at lower levels if
associated with sepsis .
* One year old infant weight 10 kg ,
arrives to the emergency room with 2 * A person weight 90 kg and height
day's history of diarrhea and poor 155 cm , BMI is :
fluid intake . Physical examination A) 27.5 kg/m2
showed pale lethargic , child heart B) 33.5 kg/m2
rate 180 / min . Low blood pressure C) 37.5 kg/m2
and 5 seconds capillary refill . D) 40.5 kg/m2
Which of the following is the most E) 45.5 kg/m2
appropriate initial fluid management
?

‫اسئلــة االطفـــال‬ ‫ اسامه الخزاعله‬: ‫الدكتور‬


Page 29
A) Turner syndrome
B) Chronic renal failure
C) Small for gestational age
D) Prader Willi syndrome
* A 18 month old child presented E) Achondroplasia
with chronic diarrhea , poor weight
gain and iron deficiency anemia ,
his brother has type one diabetes
mellitus . All of the following
regarding this case are true
except ?
A . Screening test is IgA anti * Child with SCA , the vaccine must
tissue transglutaminase . be given post autosplenectomy is :
B . Screening his brother with DM A) DTap vaccine
for the disease is indicated . B) MMR vaccine
C . Patient can return to normal C) Hib vaccine
usual diet one anti TTG . D) Pneumococcal vaccine
D . Decreasing titer of anti TTG E) BCG vaccine
reflect complains with celiac
disease . * All are autosomal recessive
E . Normal IgA anti TTG does not disease except :
rule out of the disease . A) Cystic fibrosis
B) B thalassemia
* Your college sends you a patient C) PKU
with an accompanying note _ please D) Marfan syndrome
assess this child who has a grade 5 E) SCA
_ 6 murmur . What associated
physical finding would you expect ? * How long does expressed fresh
A . Silent chest . breast milk keep at room temperature
B . Thrill . :
C . Heave . A) 3 hours
D . Precordial bulge . B) 6 hours
E . Dextrocardia . C) 8 hours
D) 12 hours
* A 4 years old child presents with
profuse watery diarrhea which is * How many cm should a child grow
positive for occult blood . She is per year between the age 2_12
febrile to 38.8 and her WBC is years :
19.000 . She is currently completing A) 3 cm /year
a 7 day course of amoxicillin B) 6 cm /year
clavulanic for an ear infection . C) 9 cm /year
Which of the following is the most D) 12 cm /year
appropriate therapy at this time
would be ? * One of the following are
A . Vancomycin . contraindications for breast feeding
B . SC vancomycin . :
C . IV metronidazole . A) Chloramphenicol therapy
D . PO metronidazole . B) Ceftriaxone therapy
E . PO clindamycin . C) Mastitis
D) Breast feeding jaundice
Rice watery diarrhea is
characteristic for : * The average head circumference
A) Rota virus growth in first 6 month of life is :
B) E coli A) 3 cm
C) Cholera B) 6 cm
D) Cryptosporidiosis C) 9 cm
E) Clostridium D) 12 cm

* Human growth hormone is effective * All of the following is autosomal


in all except : recessive disease except :

‫اسئلــة االطفـــال‬ ‫ اسامه الخزاعله‬: ‫الدكتور‬


Page 30
A) Achondroplasia * In 18 months old child can do all
B) Phenylketonuria except :
C) Galactosemia A) He runs well
D) CAH B) Copies a circle
C) Do tower of 3 cubic
* All of the following has airborne D) Eats with spoon
mode of transmission except :
A) Measles * About G6PD all are true except :
B) Varicella A) X linked recessive disease
C) Scarlet fever B) It has neonatal screening test
D) Hand ,foot and mouth disease C) Reticulocyte count is low in
acute crisis
D) In neonate present as prolonged
pathological jaundice
* One of the following is water
borne disease : * All are seen in Rheumatic fever
A) Hepatitis C except :
B) HIV A) Short PR interval on ECG
C) Cholera B) CRP elevated
D) Malaria C) Sydenham chorea
D) Pancarditis E) Subcutaneous
* All are seen in Kawasaki disease nodules
except : * All of the following are major
A) Peak age 3 months to 5 years criteria rheumatic fever except :
B) Fever persisting 5 days A) Sydenham chorea
C) Most severe complication is B) Migratory arthritis
splenic rupture C) Rheumatic nodules
D) Most common early complication is D) Erythema nodosum
Myocarditis E) Pancarditis

* One of the following is a cause of * All are seen in infective


Hyposplenism : endocarditis except :
A) Gluten enteropathy A) Janeway lesions
B) Sarcoidosis B) Roth spots
C) Malaria C) Osler nodes
D) Megaloblastic anemia D) Erythema marginatum
E) Splinter hemorrhage
* MCV value in Macrocytic anemia
is : * All are causes of metabolic
A) More than 60 Fl alkalosis except :
B) More than 70 Fl A) Pyloric stenosis
C) More than 90 Fl B) Furosemide
D) More than 100 Fl C) Bartter syndrome
E) More than 120 Fl D) Addison disease
E) Gitelman syndrome
* All are true about breast feeding
except : * All are causes of respiratory
A) Easily digested and low renal acidosis except :
solute load A) Narcotics
B) Contains IgG antibodies B) Kyphoscoliosis
C) Ca :Po4 ratio 1.8:1 C) Diaphragm muscle paralysis
D) Contains protein 0.9_1.5% D) Hyperkalemia
E) Severe bronchial asthma
* 3 years old child can do all of
the following except : * One is true about Hypernatemia :
A) Can draw square A) Correction of serum Na 1 mmol
B) Rides tricycle /hour
C) Climbs upstairs non alternating B) Na level > 160 mmol/L considered
D) Combines 3 words sentence Hypernatemia
C) More common than Hyponatremia

‫اسئلــة االطفـــال‬ ‫ اسامه الخزاعله‬: ‫الدكتور‬


Page 31
D) SIADH is one of the causes C) Can effectively be treated by
E) Associated with CHF Albendazole
D) It is protozoon infection
* The normal range of plasma
osmolality is : * About ITP one is false :
A) 265_280 mosm /kg A) Autoimmune disease
B) 280_295 mosm / kg B) Bruises and Petechiae on the
C) 295_305 mosm / kg limbs
D) 305_320 mosm / kg C) Lymphadenopathy
D) Best initial treatment is
* Regarding treatment of Acute steroids
Epiglottitis , the best first step E) In chronic cases treated by
is : splenectomy
A) IV antibiotics
B) Humidified oxygen by SFM * All are causes of hypoglycemia
C) Corticosteroids except :
D) Intubation A) Growth hormone deficiency
E) Salbutamol B) Alcohol consumption
C) Liver cirrhosis
* Best for diagnosis of Achalasia in D) Pheochromocytoma
children : E) Pituitary failure
A) 24 hours esophageal PH metry
B) 24 hours esophageal manometry
C) Esophago_Gastro_Duodenoscopy * A type of abdominal wall defect in
D) Barium swallow which the intestine , liver , and
other organs remain outside of the
abdominal cavity without sac due to
defect in development of the Muscles
* 30 months old baby bring to you in of the abdominal wall is called :
E.R . C/O dyspnea , Tachypnea , A) Umbilical hernia
hypoxia , nasal flaring , muscle B) Exomphalos
retraction , barking cough , x Ray C) Inspissation
shows a narrow subglottic airway D) Gastroschisis
( Steeple sign ) , what is the
likely diagnosis :
A) Acute Epiglottitis * Pre term infant at high risk of
B) Croup all of the following except :
C) Bacterial Tracheitis A) Broncho_pulmonary dysplasia
D) Herpangina B) Meconium aspiration syndrome
E) Laryngospasm C) PDA
D) NEC
* All are causes of macrocephaly
except : * 3 month old infant presented with
A) CMV infection watery stool ,no fever , no blood in
B) Neurofibromatosis type one stool , baby is poor eating , the
C) Tuberous sclerosis best next step is :
D) Fragile x syndrome A) Admission and give him motilium
E) Sotos syndrome B) Admission and do stool analysis
C) Outpatient treatment with ORS
* Which of the following is common D) Admission and give oral or IV
in all forms of shock : rehydration
A) Decreased cardiac output E) Admission and give him
B) Increased heart rate antibiotics
C) Decreased CVP
D) Decreased tissue perfusion * The best test for evaluating cases
of meconium ileus is :
* Each of the following are true A) Contrast enema study
about Giardia lamblia except : B) Ultrasonography
A) It is transmitted feco oral route C) Plain abdominal X_Ray
B) It causes chronic diarrhea D) Pancreatic function test

‫اسئلــة االطفـــال‬ ‫ اسامه الخزاعله‬: ‫الدكتور‬


Page 32
E) Liver disease
* All are true about Crohn disease
except : * Least common valvular heart
A) Most common site is terminal disease lead to infective
ileum endocarditis :
B) Skip irregular lesions A) Aortic stenosis
C) Lead pipe like of abdominal X_Ray B) Mitral stenosis
D) Slight increase risk of colon C) Tricuspid stenosis
cancer D) Pulmonary stenosis

* All are benign neonatal skin * Least common congenital heart


conditions except : disease with left to right shunt
A) Cutis marmorata that can lead to infective
B) Erythema toxicum endocarditis :
C) Salmon patch A) VSD
D) Erythema herpetiformis B) ASD
E) Pustular melanosis C) PDA
D) ECD
* Select the correct statement
regarding the GCS * Most common congenital heart
A) A score of 3 is normal disease with left to right shunt
B) A score of 7 represents coma that can lead to congestive heart
C) A score of 11 represents severe failure :
head injury A) ASD
D) A score of 8 represents mild head B) PDA
injury C) TOF
E) A score of 15 represents bad D) VSD
prognosis E) TGV

* Most common solid tumor in * All are used for treatment of GERD
children : except :
A) Wilms tumor A) Omeprazole
B) Brain tumor B) Metoclopromide
C) Ewing sarcoma C) Frequent and smaller feeds
D) Neuroblastoma D) Ranitidine
E) Loratidine
* Normal child start to walk without
support at : * The first drug of choice in case
A) 9 month of organophosphours poisoning :
B) 12 month A) Benzodiazepines
C) 15 month B) Activated charcoal
D) 18 month C) Atropine
D) Protamine

* All are true about complex partial


seizures exc
A) Automatisms
* All are Aminoacids except : B) Can be come generalized
A) Arginine C) Loss of conscious or altered one
B) Methionine D) Illusions and hallucinations
C) Tryptophan E) No aura or post ictal period
D) Keratine * All are true about simple partial
E) Cystine seizures except
A) Some patients have an aura
* All of the following are causes of B) No post ictal period
Macrocytosis except : C) Loss of consciousness
A) Megaloblastic anemia D) On EEG multi focal spike
B) Hypothyroidism E) Phenytoin or Carbamezipine used
C) Methotrexate therapy for treatment
D) Lead toxicity

‫اسئلــة االطفـــال‬ ‫ اسامه الخزاعله‬: ‫الدكتور‬


Page 33
* First step in management of DKA A) Prosthetic heart valves
coma : B) Isolated secundum ASD
A) Start insulin infusion C) TOF
B) Start saline IV infusion D) Uncorrected aortic stenosis
C) Start IV antibiotics E) MVP with regurgitation
D) Airway scuring
E) Start bicarbonate IV * All are true about Retinopathy of
prematurity except :
* All are true about IDA except : A) It is a major cause of blindness
A) Most common cause of childhood in developed countries
anemia B) Higher risk premature and low
B) Common diagnosis between 6 months birth weight < 1500 gram
_ 3 y C) Retinal detachment is seen in
C) Pica is a sign of IDA stages 2 & 3
D) Low serum Ferritin and TIBC D) High o2 exposure after birth is
E) Hypochromic Microcytic risk factor

* All are causes of Hyperkalemia * All are true about severe life
except : threatening asthma except :
A) Tumor lysis syndrome A) Altered consciousness level
B) Conn syndrome B) Cyanosis
C) CAH C) FEV1 50_70%
D) Addison disease D) Silent chest
E) Respiratory acidosis
* Aldosterone secretion from :
A) Zona Reticularis * All Drugs are used for treatment
B) Zona Fasciculata of Acute attack of bronchial asthma
C) Medulla except :
D) Zona Glomerulosa A) Salbutamol
E) Juxtaglomerular apparatus B) Ipratropium
C) Inhaled steroids
* Hashimoto thyroiditis associated D) Adrenaline
with all of the following except : E) Aminophylline
A) DM type one
B) Down syndrome * One of the following vaccines is
C) Thyroid lymphoma contraindicated in child with Burton
D) Cushing disease agammaglobulinemia :
A) DTap vaccine
*All are true about Nephrotic B) MMR vaccine
syndrome except : C) Hepatitis B vaccine
A) MCD is the most common cause D) Pneumococcal vaccine
B) Hypoalbuminemia E) Hib vaccine
C) Macroscopic Hematuria
D) No response to steroids * All are true about Salicylates
indication for renal biopsy toxicity except :
E) Massive Proteinuria > 40 mg/m2/hr A) Early stage cause
Hyperventilation and respiratory
* All are true about ASD except : alkalosis
A) Ostium primum is the most common B) Late stage cause metabolic
type acidosis and Hypokalemia
B) Usually close spontaneously if < C) Hyperthermia is an indication of
8mm severe toxicity
C) Usually asymptomatic in childhood D) Nausea ,vomiting , diaphoresis
D) On physical wide split of S2 and tinnitus are the earliest signs
E) Plethoric lung fields on CXR and symptoms
E) N acetylcysteine is the antidote
* All of the following conditions
require prophylactic antibiotics to
prevent infective endocarditis
except :

‫اسئلــة االطفـــال‬ ‫ اسامه الخزاعله‬: ‫الدكتور‬


Page 34
C . Metabolic acidosis fully
compensated .
* All are true about developmental D . Respiratory acidosis
dislocation of the HIP except : uncompensated .
A) There is screening program for E . Metabolic Alkalosis fully
DDH compensated
B) It usually affect right hip
C) It more common in first born * Regarding febrile seizures , all
female are true except ?
D) Breech presentation is risk A . The most common cause of
factor seizures in children .
E) Dynamic ultrasound is the gold B . Occurs between 6 months and 6
standard for diagnosis in first 6 years .
month of life C . Lumbar puncture is not done
routinely in all children .
* All are signs and symptoms of D . Is a recognized cause of status
cystic fibrosis except : epilepticus in children .
A) Most early sign is meconium ileus E . Sodium valproate is given after
B) Pancreas insufficiency the second attack to prevent
C) It has screening program in recurrence .
Jordan * One of the following is false
D) Finger clubbing regarding poststreptococcal
E) Infertility later in life glomerulonephritis ?
A . Urine analysis usually shows
* All are true about FMF except : RBCs , RBCs casts , and mild
A) 90 % of patients have thier first proteinuria .
attack before the age of 18 years B . Early antibiotic treatment for
old Streptococcal infection will
B) Common first attack are fever eliminate the risk of
with abdominal pain , joint pain , glomerulonephritis .
chest pain , C . Complete recovery occurs in most
muscle pain or Erysipeloid patients .
C) Complicated by Amyloidosis and D . Commonly follows Streptococcal
kidney failure pharyngitis after 2 _ 3 weeks .
D) Mutation in the chromosome 6 E . Typically presents with sudden
E) Colchicine decreases attack Onest of dark urine , edema , and
frequency proteinuria , HTN .

* All of the following are true * Charcoal is ineffective in all of


hepatitis B except : the following poisoning except ?
A) All hepatitis B carries have A . Corrosive agents .
antibodies to HBcAg and HBeAg B . Kerosin .
B) Hepatitis B carries with high C . Iron .
HBeAg are more likely to have liver D . Lead .
damage E . Aminophylline .
C) It is RNA virus
D) Hepatitis B vaccine presents in * A 4 year old girl has labial
Jordan vaccination program adhesions , which of the following
is true ?
* 3 years old child has had diarrhea A . It is not related to hygiene .
for past 3 days . Respiratory rate B . Most commonly occurs in girls
is elevated and the fontanelles are above 12 years of age .
sunken . The results of ABGs show PH C . No treatment is necessary if the
= 7.39 , PCO2 = 30 mmhg , HCO3 = 19 patient has normal urinary flow .
meq/L . Most likely acid_base D . Treatment includes the forceful
disorder ? separation of the adhesions .
A . Respiratory Alkalosis fully E . A progesterone containing cream
compensated . can be used to treat these adhesions
B . Metabolic acidosis uncompensated .
.

‫اسئلــة االطفـــال‬ ‫ اسامه الخزاعله‬: ‫الدكتور‬


Page 35
* A child who is found collapsed in E . Aniridia and Wilms tumor
the playground is brought to the
emergency room . She opens eyes to * A 4 year old boy presents with
pain stimulation , she has no verbal patchy hair loss and a scaly itching
response , and she withdraws her scalp , what is the most likely
arms from pain , the GCS of the diagnosis ?
patient is ? A . Tinea capitis .
A . 7 B . Trichotillomania .
B . 3 C . Traction alopecia .
C . 9 D . Alopecia areata .
D . 11 E . Staphylococcus scalded skin
E . 14 syndrome .

* Regarding allergic reactions , * All of the following are commonly


which of the following associations seen in patients with HSP except ?
is true ? A . Hematuria .
A . Skin prick testing effective for B . Arthritis .
diagnosis of all types of C . Purpura .
hypersensitivity . D . Abdominal pain .
B . Desensitization is an effective E . Thrombocytopenia .
treatment for Steven Johnson
syndrome . * A person will be traveling to a
C . Serum sickness can occur in malaria endemic area in the middle
response to a drug or viral east , the best prophylactic
infection . intervention is ?
D . Anaphylactic children should A . Prophylactic malaria vaccine .
receive long term corticosteroids . B . Chloroquine .
E . Hypersensitivity reactions C . Mefloquine .
always occur within minutes of D . Doxycycline .
exposure to allergen . E . Primaquine .

* 12 years old student has had


* All of the following medications vomiting and decreased level of
are not recommended in infant except consciousness . Student displays
? slow and deep breathing , he appears
A . Ciprofloxacin . dehydrated , and had a two week
B . Acyclovir . history of polydipsia , polyuria and
C . Tetracycline . weight loss . ABG shows PH = 7.0 ,
D . Cough syrup . PO2 = 90 mmhg , PCO2 = 23 mmhg ,
E . Antiemetics . HCO3 = 12 mmol/L . Lab's show Na =
126 mmol /L , K = 5 mmol /L , CL =
* Calculate the first hour fluid 95 mmol/L .
management for a 30 kg 8 years old Most likely acid_base disorder ?
child with first degree burn 10% A . Respiratory acidosis
body surface area and second degree uncompensated .
burn 10% body surface area ? B . Respiratory acidosis compensated
A . 50 cc . .
B . 75 cc . C . Metabolic acidosis compensated
C . 100 cc . partially .
D . 150 cc . D . Metabolic acidosis fully
E . 200 cc . compensated .
E . Metabolic Alkalosis
* Which one of the following compensated .
associations is wrong ?
A . Lymphoma and neck mass . * Gower sign is characteristic for ?
B . Burkitt lymphoma and EBV . A . Prader willi syndrome .
C . Reed Sternberg cell and Hodgkin B . Duchenne muscular dystrophy .
lymphoma . C . Angelman syndrome .
D . Hemihypertrophy and D . Noonan syndrome .
neuroblastoma . E . Klinefelter syndrome .

‫اسئلــة االطفـــال‬ ‫ اسامه الخزاعله‬: ‫الدكتور‬


Page 36
C . No smiling at 3 months .
* Most common genetic syndrome D . No pointing at 18 months .
associated with advanced maternal E . No head support at 5 months .
age is ?
A . Patau syndrome . * Normally child responds to own
B . Edward syndrome . name and separation anxiety begins ,
C . Down syndrome . mature pincer grasp . At which age ?
D . Turner syndrome . A . 6 months .
E . Fragile x syndrome . B . 9 months .
C . 12 months .
* All of the following are causes of D . 15 months .
non bilious vomiting in neonates E . 18 months .
except ?
A . Pyloric stenosis . * All of the following
B . GERD . investigations are initial tests to
C . Sepsis . diagnose the cause of short stature
D . Duodenal atresia . except ?
E . Tracheosesophageal fistula . A . Antibodies for celiac disease .
B . Thyroid function test .
* Most common cyanotic heart disease C . Kidney function test .
diagnosed in neonatal period is ? D . Liver function test .
A . Tetralogy of fallot . E . Growth hormone .
B . Ebstein anomaly .
C . Tricuspid atresia . * The most common solid tumor in
D . Truncus arteriosus . children is ?
E . Transposition of great vessels A . Neuroblastoma .
B . CNS tumor .
* All are cyanotic congenital heart C . Retinoblastoma .
disease except D . Wilms tumor .
A . Truncus arteriosus . E . Bone tumors .
B . Tricuspid atresia .
C . Ebstein anomaly . * Breastfed infants should be
D . Coarctation of aorta . supplemented with
E . Tetralogy of fallot . A . Vitamin A
B . Vitamin B
* All of the following drug's can C . Vitamin C
cross into breast milk except ? D . Vitamin D
A . Bromocriptine . E . Vitamin E
B . Warfarin .
C . Gold . * All are risk factors for
D . Tetracycline . Kernicterus except ?
E . Metronidazole . A . Hypoglycemia .
B . Sepsis .
C . Gestational age more than 38
weeks .
D . Hemolysis .
* The most likely organism to cause E . Acidosis .
osteomyelitis in an 8 years old boy
is ?
A . Staphylococcus aureus .
B . Kingella kingae .
C . Streptococcus pneumoniae .
D . Streptococcus pyogenes .
E . Haemophilus influenzae type B . * All of the following can cause
microcytic hypochromic anemia except
* All are developmental red flags ?
except ? A . Hypothyroidism .
A . No walking at 9 months . B . Lead toxicity .
B . Less than 10 words at 18 C . Iron deficiency .
months . D . Thalassemia .

‫اسئلــة االطفـــال‬ ‫ اسامه الخزاعله‬: ‫الدكتور‬


Page 37
E . Sideroblastic anemia . D . Start oxygen .
E . Discharge home and tell mother
* All of the following regarding that it is self limiting .
cystic fibrosis are true except ?
A . Is an autosomal recessive . * National neonatal screening in
B . May present with meconium ileus Jordan includes A . Congenital
at neonatal period . adrenal hyperplasia , hypothyroidism
C . Is diagnosed by sweat chloride , Phenylketonuria and G6PD .
test . B . Hypothyroidism , Galactosemia ,
D . Nasal polyps are more common Phenylketonuria and G6PD .
with it . C . Hypothyroidism and
E . All case's have exocrine Phenylketonuria .
pancreatic insufficiency . D . Hypothyroidism , Phenylketonuria
and Thalassemia .
* A 2 week old baby presented to E . Hypothyroidism , G6PD deficiency
your clinic with history of poor and Phenylketonuria
feeding and decreased activity in
the past 2 day's , the best next * The following is true regarding
step is ? Erythema infectiosum except ?
A . To give IV fluid to treat A . Caused by HHV type 6 or 7 .
dehydration . B . Teratogenic .
B . To give oral strong C . Usually causes pancytopenia in
antibiotics . patient with thalassemia .
C . To admit him to the hospital . D . Not contagious once the rash
D . To send him home and see him on erupts .
the second day after taking blood E . Serious disease and needs
culture . aggressive treatment .
E . To send him for observation and
to feed him well by breastfeeding . * Cryptorchidism is characteristic
by which of the following ?
* Regarding congenital primary A . Surgical intervention is always
hypothyroidism . One of the required .
following is true ? B . In true Cryptorchidism , many
A . Half of the case's have no testes will often descends
symptoms at birth . spontaneously .
B . The most common cause is C . In ectopic Cryptorchidism , the
dysgenesis . testes can be milked down the
C . Treatment is with T3 in the scrotum .
permanent case's D . In ectopic Cryptorchidism , the
D . TSH and T4 are low . testicles should be pinned to the
E . It is inherited mostly as an scrotum
autosomal recessive disease before the age of puberty .
E . Retractile testicles are
* Urinary VMA and HVA can be used associated with congenital adrenal
for diagnosis of ? hyperplasia .
A . Wilms tumor .
B . Neuroblastoma . * How much sodium in meq in half
C . Neurofibromatosis . liter normal saline ?
D . Acute lymphocytic leukemia . A . 50 .
E . Horner syndrome . B . 77 .
C . 130 .
* You are about to discharge a term D . 154 .
newborn but notice the baby's skin E . 65 .
have red spots with overlying white
papules , heart rate 150 BPM and no * A 5 year old girl has three months
murmur on exam . What is the next of painful swelling of both knees .
step in management ? Her parents said that she complains
A . Head ultrasound . of morning stiffness and her gait
B . Initiate prostaglandin therapy . like an old man . What is the most
C . IV fluid . likely diagnosis ?

‫اسئلــة االطفـــال‬ ‫ اسامه الخزاعله‬: ‫الدكتور‬


Page 38
A . Juvenile idiopathic arthritis . * One is true about caustic
B . Septic arthritis . ingestion ?
C . Post infectious arthropathy . A . Early endoscopy is indication .
D . Systemic lupus erythematosus . B . Induce vomiting reduce the
E . Scleroderma . complications .
C . Absence of the mouth lesions
* Children are usually expected to exclude the esophageal injury .
walk at age ? D . Gastric lavage is
A . 6 months . contraindicated .
B . 9 months . E . Acidotic agent's is more harmful
C . 12 months . than the alkaline agent's .
D . 18 months .
E . 2 years . * All are types of cerebral palsy
* A 6 month old boy has had a fever except ?
up to 39.4 for the past 8 day's with A . Hemiplegic .
excretions conjunctivitis , red lips B . Quadriplegic .
, cervical lymph nodes enlargement C . Atonic .
and sore throat on examination . All D . Infantile spasm .
of the following would be an E . Extrapyramidal
appropriate part of the management
except ?
A . Echocardiogram .
B . High dose aspirin .
C . IV immunoglobulin .
D . Antipyretic therapy .
E . Serial abdominal ultrasound * In the presence of symptoms
diabetes mellitus is diagnosed when
* Regarding immunodeficiency , which random blood sugar exceed ?
of the following associations is A . 100 mg/dl .
false ? B . 125 mg/dl .
A . IgA deficiency associated with C . 140 mg/dl .
celiac disease D . 200 mg/dl .
B . Common variable immunodeficiency E . 110 mg/dl .
_ there is no vaccine response in
the body . * Which of the following vaccines is
C . Bruton agammaglobulinemia _ part of the Jordanian national
there are no B cells . vaccination program ?
D . Digeorge syndrome _ X linked , A . Meningococcal conjugate
triad of eczema , low platelets and vaccine .
severe suppurative skin lesions . B . Pneumococcal conjugate vaccine .
E . Leukocyte adhesion defect _ C . Varicella vaccine .
delayed separation of umbilical D . Hepatitis A vaccine .
stump . E . Inactivated polio vaccine .

* Regarding idiopathic nephrotic * Of all the features of Tetralogy


syndrome in children , all of the of fallot , which of the following
following are true except ? is the best index for prognosis ?
A . Influenza vaccine should be A . Size of right ventricle .
given on a yearly basis . B . Aortic size .
B . The majority of children with C . Severity of pulmonary stenosis .
steroid responsive N.S have repeated D . Sex of the baby , boy's have
relapses . worse prognosis than girls .
C . Renal biopsy should be done for E . Size of the VSD .
the diagnosis in most case's .
D . Peritonitis is a complication of * All of the following are true
nephrotic syndrome . about VSDs except
E . MCD is the most common A . Results in left to right shunt .
histological lesion . B . The primary murmur is systolic .

‫اسئلــة االطفـــال‬ ‫ اسامه الخزاعله‬: ‫الدكتور‬


Page 39
C . Membranous VSDs in the upper D . Mitochondrial
part of the septum are more likely E . x-linked dominant
to close spontaneously .
D . Large VSDs are more likely to * AIl of the followings could be
present with less murmur and early options for treating Brucella except
heart failure . ?
E . Cyanosis is not a common feature A . Doxycycline.
. B . Rifampicin.
C . TMP/SMX. TMP SMX.
* Which of the following is more D . Gentamicin.
indicative of a pathologic murmur E . Cefazolin
rather than innocent murmur ?
A . The murmur is heard during * AIl of the followings are we
diastole . attenuated vaccines except ?
B . The child has no other A . Rotavirus
symptoms . B . Varicella
C . The murmur is soft ( grade 2 or C . Measles
lower ) D . Oral Polio Vaccine
D . The CXR is normal . E . Hepatitis
E . The murmur does not radiate to
the back . * AIl of the following about BCG
vaccine are true except ?
* A 9 year old child presents with A . It is included in the Jordanian
abdominal pain , headache . His national immunization program .
urine is positive for ketones and B . It has little effect on the
sugar . All of the following are reactivation disease .
appropriate during C . May affect TB skin test (PPD)
management except ? result. (PPD) .
A . Insulin bolus of 0.1 units per D . Can be given from birth .
kilogram . E . It is a killed bacterial vaccine
B . Keep child NPO . .
C . IV bolus of 20 ml/kg of normal
saline . * Central precocious puberty in boys
D . Monitor serum potassium . starts with inaeased testicular
E . Admit to the ICU to observe . volume before the age of ?
A . 7 years
* The IV fuid maintenance for a 15 B . 8 years
kg child is: C . 9 years
A . 750 cc . D . 10 years
B . 1250 cc . E . 6 years
C . 1900 cc .
D . 2500 cc . * In a 5 year old child with
E . 3000 cc . osteomyelitis, the most common
* patients with cystic fibrosis, al causative organism is ?
of the folowing are expected A . Staphylococcus aureus
except ? B . Escherichia coli.
A . Rectal prolapse C . Group A streptococcus
B . Nasal polyp D . salmonella typhi
C . Core pulmonale E . Haemophilus Influenza type B
D . Azoospermia as an adult
E . Aneurysm of coronary arteries * In DKA all are true except ?
A . insulin should not be given iv
* In an inherited disease, the males bolus
and females are equally affect . The B . PH is below 7.3 and or
risk of siblings having the disease Bicarbonate is less than 15mg/ dl .
is 25% if both parents are carriers. C . Positive ketone in serum and
This disease is inherited as ? urine .
A . Autosomal dominant D . Nacl solution should be given
B . Autosomal recessive initially .
C . X-Linked recessive

‫اسئلــة االطفـــال‬ ‫ اسامه الخزاعله‬: ‫الدكتور‬


Page 40
E . NaHCO3 is given after 2 hours of * The therapy that can remove the
starting treatment . potassium from the body ?
A . 10% calcium gluconate .
* AIl of the following parameters B . Bicarbonate .
are used to monitor infant baby with C . Nebulized albuterol .
hypovolemic shock except ? D . Insulin .
A . Level of consciousness . E . sodium polystyrene sulfonate
B . Capillary refill time . (Kayexalate)
C . Pupillary reaction .
D . Blood pressure . * One of following diseases can
E . Skin turgor . cause anemia in the child with
hemoglobinopathy ?
* All of the following are A . Roseola Infantum (Sixth disease)
recognized causes of hemolytic B . Erythema Infectiousum (Fifth
disease in the newborn except ? disease)
A . Hereditary spherocytosis . C . Hand Foot Mouth disease.
B . G6PD . D . Chicken pox.
C . Sickle cell disease . E . Measles
D . ABO incompatibility .
E . Rh incompatibility . * neonatal seizures may present by
all of the following pictures except
* The most common type of cerebral ?
palsy ? A . Eye blinking
A . atonic B . Apnoea
B . Ataxic C . Nystagmus
C . Spastic D . sucking
D . Hypotonic E . jitteriness
E . Mixed
* Charcoal is ineffective in all of
* one year old boy brough to the ER the following poisoning except ?
for an attack of generalized tonic A . Corrosive agentsas
seizure, In the management of his B . Kerosin
seizures all are true except ? C . Iron
A . Serum glucose, calcium and D . lead
sodium should be measured . E . aminophylline
B . First line of treatment with
anti epileptic drugs is * 10 month old child can do all of
benzodiazepine . the following except ?
C . Phenytoin is given iv over 20 A . sits without support, with back
minute . straight .
D . when the cause is hypoglycemia B . Pulls to stand position walks on
2cc/kg of Dextrose 10% is given iv . to furniture .
E . the calcium gluconate is given C . Grasps objects with thumb and
intramuscular when its serum level forefinger.
is low . D . Says 3-5 words specifically .
E . wave bye bye .
* 11-month-old girl had an illness
characterized by fever up to 39.40C * Four week old male infant
mid imitabity for 3 days. then presented with tiredness and rapid
subsided but folowed by a smal pink breathing during feeding, he has to
macules over the trunk and The stop sucking frequently to catch his
patient most key has ? breast -Which one of the following
A . Roseola Infantum . is Most appropriate investigation?
B . Pitryasis Rosea . A . Complete blood count .
C . Measles . B . Echocardiography E .
D . scarlet fever . C . electroencephalography .
E . Erythema infectiosum . D . Arterial blood gas .
E . Serum electrolytes .

‫اسئلــة االطفـــال‬ ‫ اسامه الخزاعله‬: ‫الدكتور‬


Page 41
A . Carditis
B . Arthralgia
C . Erythema marginatum
D . Sydenham's chorea
E . Shortened PR interval .

* Regarding idiopathic * The cerebrospinal fluid (CSF) of 3


thrombocytopenia purpura , all of year old chad wth suspected
the followings are true except ? meningitis showed WBC 150,
A . it is the most common cause of neutrophies 10%, protein CSF glucose
acute onset of thrombocytopenia in 90 while blood glucose 100. is most
otherwise healthy . likely consistent with ?
B . The peak age is 1-4 years of age A . Bacterial Meningitis .
. B . Early viral meningitis .
C . Usually preceded by viral C . TB meningitis .
infection 1-4 weeks before the onset D . Fungal meningitis .
. E . Normal csF analysis for age .
D . Platelets size is normal or
increased . * The initial investigation for
E . Spleen is enlarged in majority short stature includes all except ?
of the cases A . CBC .
B . Thyroid function test .
* Regarding immune deficiency, all C . Growth hormone .
of the following associations are D . Kidney function tests .
true except ? E . Bone age .
A . Hypocalcemia and DiGeorge
syndrome . *All of the folowing are recognized
B . Delayed umbilical cord as normal findings in a newborn baby
detachment and leukocyte adhesion except ?
defect . A . Mongolian blue spot on low
C . Recurrent Neisseria meningitis spinal area .
and hyper IgM syndrome . B . Group of vesicles in close
D . Severe progressive infectious proximity on scalp .
mononucleosis and X- linked syndrome C . vaginal blood spotting .
. D . Lanugo hair on extremities .
E . Recurrent cutaneous abscesses E . Erythema toxicum on trunk .
and hyper lgE syndrome .
* All of the following association
* All of the following can ocour in regarding genetic disorders are true
patients with Kawasaki disease except ?
except ? A . Down syndrome and atlantoaxial
A . Coronary artery aneurysm. sublaxation .
B . Aseptic meningitis. B . Turner syndrome and aortic valve
C . Hydrops of the gall bladder anomalies .
D . Osteomyelitis C . Allagile syndrome and
E . sterile pyuria cholestatic jaundice .
D . Klinefelter syndrome and short
* Blood that is used for transfusion stature .
purposes is routinely for serologic E . Pradder willi syndrome and
markers for all of the following obesity .
diseases except ?
A . HBC (hepatitis B virus ) . * Regarding urinary tract infection
B . HSV (herpes simplex virus ) . in children, one of the pathogens to
C . syphilis . the formation of phosphate stones by
D . CMV (cytomegalovirus) . splitting urea to ammonia in the
E . AIDS . urinalysis ?
A . Escerichea Coli
* one of the folowing is not B . Pseudomonas
considered as Jones criteria for C . Streptococcus fecalis
rheumatic fever ? D . Proteus

‫اسئلــة االطفـــال‬ ‫ اسامه الخزاعله‬: ‫الدكتور‬


Page 42
E . Staphylococcus
* The most common hereditary
* You are attending the birth of an bleeding disorder
infant whose Mather smoked 2 packs A . Hemophilia A .
of cigarettes per day during the B . Hemophilia B .
pregnancy.which of the followings C . Von Willebrand disease
finding would be most likely D . DIC .
presented in the newborn ? E . Idiopathic Thrombocytopenic
A . growth retardation . purpura .
B . Hydronephrosis .
C . Microcephaly . * The leading cause of death among
D . Respiratory distress syndrome . school children is ?
E . Hypoglycemia . A . congenital anomalies
B . Injuries
* Acrodermatis enteropathica is C . Malignant disease
treated with ? D . Communicable disease
A . Vitamin A . E . Respiratory illnesses
B . Zinc .
C . Phosphate . * Five year male, known case
D . Magnesium . frequent relapsing nephrotic
E . Vitamin B2 . syndrome on steroid treatment. This
patient is at risk for all the
following except ?
A . peritonitis
B . Malnutrition
C . Bleeding tendency
D . Hyperlipidemia
E . immunosuppression
* Regarding congenital
hypothyroidism one is true ?
A . most of the cases are
secondary .
B . All cases must be treated with
thyroxin .
C . Treatment is usually stopped
after8 months .
D . Most of the cases are familial . * One week old newborn presented
E . symptoms seen by the end of the cholestatic jaundice ; hypoglycemia
first week. and seizure . You are expecting
galactosemia as a diagnos, the gold
* A 5 month old infant brought by diagnostic test is ?
his mother who said that he had A . urine for reducing substances
history of vomiting 3-4 times daly. B . Measurement of GALT
His birth weight is 2.6 kg . He C . Liver biopsy
looks well with no abnormal findings D . Plasma galactose level
.his weight is 5,8 kg the best E . ophthalmology examination for
management for this case is ? catarac
A . omeprazole .
B . ranitidine .
C . metoclopramide(plasil) .
D . domperidone(motillium) .
E . no treatment is needed .

* The most common cause of childhood


meningitis in Jordan ?
A . streptococcus pneumonia.
B . Neisseria meningitides .
C . E.coli .
D . Mycobacterium Tuberculosis .
E . Staphylococcus areus .

‫اسئلــة االطفـــال‬ ‫ اسامه الخزاعله‬: ‫الدكتور‬


Page 43

You might also like